Sei sulla pagina 1di 32

Audit Report

Audit Report Answer C is incorrect because mere reading of the assertion is insufficient.

Answer D is correct because agreed-upon procedures engagements ordinarily do not require a written
1. Assurance services performed for decision makers may address the assertion.

Quality of information Context of information


A Yes Yes 4. Suitable criteria for use in an attestation engagement must be objective, measurable, complete,
B Yes No and
C No Yes
D No No Relevant Reductive
A Yes Yes
Answer A is correct because assurance services are independent professional services that improve the B Yes No
quality of information or its context for decision makers C No Yes
D No No
Answer B is incorrect. Refer to the correct answer explanation.
Answer A is incorrect. Refer to the correct answer explanation.
Answer C is incorrect. Refer to the correct answer explanation.
Answer B is correct because the attestation standards require that suitable criteria be objective,
Answer D is incorrect. Refer to the correct answer explanation. measurable, complete, and relevant—not "reductive."

Answer C is incorrect. Refer to the correct answer explanation.


2. Ordinarily in an attest examination report a CPA may report on
Answer D is incorrect. Refer to the correct answer explanation.
Management’s written assertion Subject matter
A Yes Yes
B Yes No 5. In which of the following ways may suitable criteria appropriately be made available?
C No Yes
D No No Publicly available Included with subject Included in CPA’s
matter report
Answer A is correct because in most circumstances, a CPA may report upon either the written assertion A Yes Yes Yes
or on the subject matter to which the written assertion relates. B Yes Yes No
C Yes No No
Answer B is incorrect. Refer to the correct answer explanation. D No Yes Yes

Answer C is incorrect. Refer to the correct answer explanation. Answer A is correct because suitable criteria should be available in one or more of the following ways:
(1) publicly, (2) included with the subject matter or in the assertion, (3) included in the CPA's report, (4)
Answer D is incorrect. Refer to the correct answer explanation. well understood by most users (e.g., the distance between A and B is twenty feet) or (5) available only to
specified parties.

3. Which of the following is correct concerning an agreed-upon procedure engagement performed Answer B is incorrect. Refer to the correct answer explanation.
under the attestation standards?
a) The minimum procedures to be performed include a consideration of internal control. Answer C is incorrect. Refer to the correct answer explanation.
b) The use of the report is ordinarily restricted to management.
c) Mere reading of an assertion ordinarily restricted to management. Answer D is incorrect. Refer to the correct answer explanation.
d) Mere reading of an assertion ordinarily constitutes a minimum sufficient procedure to issue a
report.
6. Which of the following are required on all attestation engagement?
Answer A is incorrect because no consideration of internal control is required unless that is what is
agreed upon. Suitable criteria Written assertion
A Yes Yes
Answer B is in correct because the use of the report is restricted to management and the specified parties B Yes No
involved. C No Yes
D No No
1
Audit Report
a) Be applied on a basis consistent with those followed in the prior year.
b) Be approved by the Auditing Standard Board or the appropriate industry subcommittee.
Answer A is incorrect. Refer to the correct answer explanation. c) Reflect transactions in a manner that presents the financial statements within a range of
acceptance limits.
Answer B is correct because while an attestation engagement will always have subject matter, it d) Match the principles used by most other entities within the entity’s particular industry.
sometimes will not have a written assertion. For example, consider a situation in which the CPA's client is
not responsible for the subject matter, in such a circumstance it may not be possible to obtain a written Answer A is incorrect because a change in accounting principle may be acceptable and appropriate.
assertion from the individual who is the responsible party.
Answer B is incorrect because the Auditing Standards Board generally does not issue accounting
Answer C is incorrect. Refer to the correct answer explanation. principles.

Answer D is incorrect. Refer to the correct answer explanation. Answer C is correct. The requirement is to determine an implication of financial statements being
presented fairly in conformity with generally accepted accounting principles. The principles selected
should reflect transactions in a manner that presents the financial statements within a range of acceptable
7. Which of the following is least likely to include a reference to the use of a specialist? limits.
a) Unqualified opinion.
b) Adverse opinion. Answer D is incorrect because the exact principles followed need not be used by most other entities
c) “Except for” qualified opinion. within the entity's particular industry.
d) “Subject to” qualified opinion.

Answer A is correct because when issuing an unqualified opinion, the auditor should not refer to the work 10. Which of the following representations does an auditor make explicitly and which implicitly when
or findings of the specialist. When an auditor decides to modify the audit opinion as a result of the report issuing an unqualified opinion?
or findings of the specialist, reference to and identification of the specialist may be made in the auditor's
report if the auditor believes such reference will facilitate understanding. Conformity with GAAP Adequacy of disclosure
A Explicitly Explicitly
Answer B is incorrect. Refer to the correct answer explanation. B Implicitly Implicitly
C Implicitly Explicitly
Answer C is incorrect. Refer to the correct answer explanation. D Explicitly Implicitly
Answer D is incorrect. Refer to the correct answer explanation. Answer A is incorrect. Refer to the correct answer explanation.

Answer B is incorrect. Refer to the correct answer explanation.


8. Miller Co. uses the first-in, first-out method of costing for its international subsidiary’s inventory
and the last-in, first-out method of costing for its domestic inventory. Under these Answer C is incorrect. Refer to the correct answer explanation.
circumstances, Miller should issue an auditor’s report with an
a) “Except for” qualified opinion. Answer D is correct. The requirement is to determine whether representations on "conformity with GAAP"
b) Unqualified opinion. and on "adequacy of disclosure" are made explicitly or implicitly in an unqualified audit report. The first
c) Explanatory paragraph as to consistency. standard of reporting requires an explicit statement on conformance with GAAP. However, the third
d) Opinion modified as to consistency. reporting standard results in implicit representations on disclosure since it states that informative
disclosures are to be considered adequate, unless otherwise stated.
Answer A is incorrect because a qualified opinion is not necessary.

Answer B is correct because the use of such differing methods may be appropriate due to the 11. Does an auditor make the following representation explicitly or implicitly when issuing the
circumstances and therefore an unqualified opinion may be issued. standard auditor’s report on comparative financial statements?
Answer C is incorrect an explanatory paragraph as to consistency is only necessary when a change in Consistent application of Examination of evidence on a test
principles has occurred. accounting principles basis
A Explicitly Explicitly
Answer D is incorrect because the opinion need not be modified. See AU 420 for information on the
B Implicitly Implicitly
consistency of application of accounting principles.
C Implicitly Explicitly
D Explicitly Implicitly
9. For an entity’s financial statements to be presented fairly in conformity with generally accepted
accounting principles, the principles selected should Answer A is incorrect. Refer to the correct answer explanation.

2
Audit Report
Answer B is incorrect. Refer to the correct answer explanation.
14. A lawyer limits a response concerning a litigated claim because the lawyer is unable to
Answer C is correct. The standard audit report implicitly assumes that accounting principles have been determine the likelihood of an unfavorable outcome. Which type of opinion should the auditor
consistently applied and explicitly states that examination of evidence was performed on a test basis. express if the litigation is adequately disclosed and the range of potential loss is material in
relation to the client’s financial statements considered as a whole?
Answer D is incorrect. Refer to the correct answer explanation. a) Adverse.
b) Unaudited.
c) Qualified.
12. How are management’s responsibility and the auditor’s responsibility represented in the d) Unqualified.
standard auditor’s report?
Answer A is incorrect because adverse opinions are used when an auditor knows that the statements are
Management’s responsibility Auditor’s responsibility misleading.
A Explicitly Explicitly
B Implicitly Implicitly Answer B is incorrect because "unaudited" is not a type of audit opinion.
C Implicitly Explicitly
D Explicitly Implicitly Answer C is incorrect. Refer to the correct answer explanation.

Answer D is correct because if the contingency has been adequately disclosed by the client, the auditor
Answer A is correct. The auditor's standard report explicitly states that the financial statements are the would issue an unqualified opinion.
responsibility of the company's management and that the auditor's responsibility is to express an opinion
on the financial statements based on his/her audit.
15. When an auditor submits a document containing audited financial statements to a client, the
Answer B is incorrect. Refer to the correct answer explanation. auditor has a responsibility to report on
a) Only the basic financial statements included in the document.
Answer C is incorrect. Refer to the correct answer explanation. b) The basic financial statements and only the additional information required to be presented in
accordance with provision of Financial Accounting Standards Board.
Answer D is incorrect. Refer to the correct answer explanation. c) All of the information included in the document.
d) Only the portion of the document which was audited.

13. Jojo, an independent auditor, was engaged to perform an examination of the financial Answer A is incorrect. Refer to the correct answer explanation.
statements of Three-R Incorporated 1 month after its fiscal year had ended. Although the
inventory count was not observed by Jojo, and accounts receivable were not confirmed by direct Answer B is incorrect. Refer to the correct answer explanation.
communication with creditors, Jojo was able to gain satisfaction by applying alternative auditing
procedures. Jojos’ auditor’s report will probably contain Answer C is correct because the professional standards require that when an auditor submits a document
a) An “except for” qualification. containing audited financial statements to his/her client or to others, s/he has a responsibility to report on
b) An unqualified opinion and an explanatory paragraph. all the information included in the document. Note that while all information is to be reported on, it need
c) Either qualified opinion or a disclaimer of opinion. not all be audited (some of it may be "unaudited").
d) A standard unqualified opinion.
Answer D is incorrect. Refer to the correct answer explanation.
Answer A is incorrect because a qualified opinion is not needed since the auditor was able to gain
satisfaction by applying alternative auditing procedures.
16. The auditor’s judgment concerning the overall fairness of the presentation of financial position,
Answer B is incorrect because an explanatory paragraph would not be required since the auditor was results of operations, and statement of cash flows is applied within the framework of
able to gain satisfaction by applying alternative auditing procedures. a) Quality control.
b) Generally accepted auditing standards which include the concept of materiality.
Answer C is incorrect because the auditor was able to gain satisfaction by applying alternative auditing c) The auditor’s consideration of the auditor company’s internal control.
procedures. No qualification or disclaimer of opinion is required when the alternative procedures allow d) Generally accepted accounting principles.
the auditor to acquire sufficient evidence regarding the inventory and accounts receivable accounts.
Answer A is incorrect because quality control relates to the policies and procedures established by a CPA
Answer D is correct because, although the auditor was not able to observe the inventory count or confirm firm to provide reasonable assurance that GAAS are being complied with on all audit engagements.
accounts receivable, the auditor was able to gain satisfaction through the use of alternative procedures.
In these circumstances, an unqualified opinion may be issued. Answer B is incorrect because GAAS relate to the conduct of audit engagements rather than the
framework within which the fairness of financial statement presentation is measured (GAAP).
3
Audit Report
c) The auditor has ascertained that Stankey’s financial statements have been prepared
Answer C is incorrect because the auditor's consideration of internal control, while important, is not as accurately.
directly related to the overall fairness of financial statement presentation as is the framework of GAAP. d) Informative disclosures in the financial statements but not necessarily in Stankey’s footnotes
are to be regarded as reasonably adequate.
Answer D is correct because the auditor's judgment concerning the fairness of financial statements
should be applied within the framework of generally accepted accounting principles. Answer A is correct because an unqualified audit report indicates that disputes over significant accounting
issues have been settled to the auditor's satisfaction. If any such disputes have not been settled, the
auditor must render an opinion other than unqualified.
17. The first standard of reporting requires that, “the report shall state whether the financial
statements are presented in accordance with generally accepted accounting principles.” This Answer B is incorrect because the objective of an audit is not to conclude that the client is financially
should be construed to require sound. Rather, the auditor renders an opinion on the fairness of the financial statement presentation in
a) A statement of fact by the auditor. accordance with GAAP.
b) An opinion by the auditor.
c) An implied measure of fairness. Answer C is incorrect because there is no assertion as to accuracy by the independent auditor. Rather,
d) An objective measure of compliance. the auditor renders an opinion on the fairness of the financial statement presentation in accordance with
GAAP.
Answer A is incorrect. Refer to the correct answer explanation.
Answer D is incorrect because the footnotes are regarded as an integral part of the financial statements
Answer B is correct because the first standard of reporting is construed to require an opinion as to and must contain adequate informative disclosures.
whether the financial statements are presented in conformity with GAAP.

Answer C is incorrect. Refer to the correct answer explanation. 20. Negative assurance is not permissible in
a) Letters required by security underwriters for data pertinent to SEC registration statements.
Answer D is incorrect. Refer to the correct answer explanation. b) Reports relating to the results of agreed upon procedures to one or more specified elements,
accounts, or items of a financial statement.
c) Reports based upon a review engagement.
18. The fourth reporting standard requires the auditor’s report to either contain an expression of d) Reports based upon an audit of the interim financial statements of a closely held business
opinion regarding the financial statements, taken as a whole, or an assertion to the effect that an entity.
opinion cannot be expressed. The objective of the fourth standard is to prevent
a) The CPA from reporting on one basic financial statement and not the others. Answer A is incorrect because letters required by security underwriters may include negative assurance.
b) The CPA from expressing different opinions on each of the basic financial statements.
c) Misinterpretations regarding the degree of responsibility the auditor is assuming. Answer B is incorrect because negative assurance is allowed for specified elements.
d) Management from reducing its final responsibility for the basic financial statements.
Answer C is incorrect because the review report issued includes negative assurance with respect to the
Answer A is incorrect because the auditor may report on one basic statement and not the other financial statements.
statements (which is not a scope limitation) as long as the auditor has access to information underlying all
the statements. Answer D is correct because, when an audit is performed, negative assurance is not to be provided,
regardless of whether the financial statements are for a fiscal or interim period.
Answer B is incorrect because the auditor may express an unqualified opinion on one of the statements
and express a qualified, adverse, or disclaimer on another if circumstances call for this treatment.
21. An auditor’s decision concerning whether or not to “dual date” the audit report is based upon the
Answer C is correct because the objective of the fourth standard of reporting (requiring an expression of auditor’s willingness to
opinion on the financial statements as a whole) is to prevent misinterpretation of the degree of a) Extend auditing procedures.
responsibility the auditor is assuming when the auditor is associated with financial statements. b) Accept responsibility for subsequent events.
c) Permit inclusion of a footnote captioned: event (unaudited) subsequent to the date of the
Answer D is incorrect because the fourth standard is not directly related to management's responsibility auditor’s report.
for the financial statements. d) Assume responsibility for events subsequent to the issuance of the auditor’s report.

Answer A is correct. If the auditor becomes aware of a subsequent event that has occurred after the
19. An investor is reading the financial statements of the Stankey Corporation and observes that the completion of fieldwork, but before the issuance of the report (which should be disclosed), the auditor
statements are accompanied by an auditor’s unqualified report. From this the investor may may dual date the report. Additionally, the auditor may date the report as of the date of the subsequent
conclude that event and extend the procedures for review of subsequent events to that date. Thus, the decision
a) Any disputes over significant accounting issues have been settled to the auditor’s satisfaction. whether or not to dual date the report is based upon the auditor's willingness to extend audit procedures.
b) The auditor is satisfied that Stankey is financially sound.
4
Audit Report
Answer B is incorrect since the auditor must accept responsibility for the known subsequent event 24. With respect to issuance of an audit report which is dual dated for a subsequent event occurring
regardless of whether dual dating is used. after the completion of fieldwork but before issuance of the auditor’s report, the auditor’s
responsibility for events occurring subsequent to the completion of fieldwork is
Answer C is incorrect because such known subsequent events are to be audited. a) Extended to include all events occurring until the date of the last subsequent event referred to.
b) Limited to the specific event referred to.
Answer D is incorrect because the auditor, with or without dual dating, need not assume responsibility for c) Limited to events occurring through the date of issuance of the report.
events subsequent to the issuance of the auditor's report. d) Extended to include all events occurring through the date of submission of the report to the
client.

22. The auditor’s report should be dated as of the date on which the Answer A is incorrect. Refer to the correct answer explanation.
a) Report is delivered to the client.
b) Fieldwork is completed. Answer B is correct because when an audit report is dual dated for a subsequent event occurring after
c) Fiscal period under audit ends. completion of fieldwork, the auditor's responsibility for events occurring subsequent to the completion of
d) Review of the working papers is completed. the fieldwork is limited to the specific event for which the report is dual dated. If the auditor dates the
report as of the subsequent event occurring after the completion of fieldwork, the auditor's responsibility is
Answer A is incorrect. Refer to the correct answer explanation. extended to all events occurring to the date of the last subsequent event.

Answer B is correct because, generally, the date of completion of the fieldwork should be used as the Answer C is incorrect. Refer to the correct answer explanation.
date of the independent auditor's report. This date is used because the auditor will be responsible for
events that occurred up to the date of completion of fieldwork. Answer D is incorrect. Refer to the correct answer explanation.

Answer C is incorrect. Refer to the correct answer explanation.


25. “Subsequent events” for reporting purposes are defined as events which occur subsequent to
Answer D is incorrect. Refer to the correct answer explanation. the
a) Balance sheet date.
b) Date of the auditor’s report.
23. Jovy, CPA, examined the 2005 financial statements of PPSI. and issued an unqualified opinion c) Balance sheet date but prior to the date of the auditor’s report.
on March 10, 2006. On April 2, 2006, Jovy became aware of a 2005 transaction that may d) Date of the auditor’s report and concern contingencies which are not reflected in the financial
materially affect the 2005 financial statements. This transaction would have been investigated statements.
had it come to Jovys’ attention during the course of the examination. Jovy should
a) Take no action because an auditor is not responsible for events subsequent to the issuance of Answer A is incorrect. Refer to the correct answer explanation.
the auditor’s report.
b) Contact PPSI’s management and request their cooperation in investigating the matter. Answer B is incorrect. Refer to the correct answer explanation.
c) Request that PPSI’s management disclose the possible effects of the newly discovered
transaction by adding an unaudited footnote to the 2005 financial statements. Answer C is correct because subsequent events are those events or transactions which occur
d) Contact all parties who might relies upon the financial statements and advise them that the subsequent to the balance-sheet date, but prior to the issuance of the financial statements and the
financial statements are misleading. auditor's report, that have a material effect on the financial statements and, therefore, require adjustment
or disclosure in the statements.
Answer A is incorrect because the auditor must take action and is responsible for the subsequent
discovery of facts existing at the date of the auditor's report. Answer D is incorrect. Refer to the correct answer explanation.

Answer B is correct. The requirement is to determine an accountant's responsibility when s/he has
become aware of a material transaction which may materially affect financial statements on which s/he 26. A major customer of an audit client suffers a fire just prior to completion of year-end fieldwork.
has already reported. The first step is to contact the firm's management and request cooperation in The audit client believes that this event could have a significant direct effect on the financial
investigation of the matter. statements. The auditor should
a) Advise management to disclose the event in notes to the financial statements.
Answer C is incorrect because the "possible effects" of this newly discovered transaction should be b) Disclose the event in the auditor’s report.
investigated further to determine whether the financials may need revision in lieu of footnote disclosure. c) Withhold submission of the auditor’s report until the extent of the direct effect on the financial
statements is known.
Answer D is impractical because it is impossible to contact all parties who might rely upon the financial d) Advise management to adjust the financial statements.
statements.
Answer A is correct because conditions which come into existence after year-end which may have a
significant direct effect on the financial statements should be disclosed in the notes to the financial
statements.
5
Audit Report
Answer B is incorrect because the auditor may or may not decide to disclose the event in the auditor's Answer C is incorrect. Refer to the correct answer explanation.
report. If the subsequent event has a material impact on the entity, the auditor may wish to include in the
report an explanatory paragraph which directs the reader's attention to the event. However, this type of Answer D is correct because when the subsequent event occurs between the date of the original report
subsequent event is only occasionally disclosed in the auditor's report. and the date of the reissuance of the report, the event may be labeled unaudited.

Answer C is incorrect because the exact total effect on the financial statements need not always be
known. The auditor cannot wait until the effect is known because this would delay the issuance of the 29. Subsequent events affecting the realization of assets ordinarily will require adjustment of the
report. financial statements under examination because such events typically represent
a) The culmination of conditions that existed at the balance sheet date.
Answer D is incorrect because only conditions coming into existence prior to year-end result in financial b) The final estimates of losses relating to casualties occurring in the subsequent events period.
statement adjustments. c) The discovery of new conditions occurring in the subsequent events period.
d) The preliminary estimate of losses to new events that occurred subsequent to the balance
sheet.
27. When a contingency is resolved immediately subsequent to the issuance of a report which was
qualified with respect to the contingency, the auditor should Answer A is correct because subsequent events affecting the realization of assets ordinarily will require
a) Insist that the client revised financial statements. adjustment of the financial statements because such events typically represent the culmination of
b) Inform the audit committee that the report cannot be relied upon. conditions that existed at the balance sheet date.
c) Take no action regarding the event.
d) Inform the appropriate authorities that the report cannot be relied upon. Answer B is incorrect because a loss related to a casualty that occurred in the subsequent events period
would not result in adjustment of the financial statements.
Answer A is incorrect because resolution of a contingency after the issuance of the report is not
subsequent discovery of facts existing at the date of the auditor's report. Since the contingency was Answer C is incorrect because the discovery of new conditions occurring in the subsequent events period
disclosed in the financial statements and audit report, the statements were not misleading at the time of would not result in adjustment of the financial statements.
issuance and do not have to be revised.
Answer D is incorrect. Refer to the correct answer explanation.
Answer B is incorrect. Refer to the correct answer explanation.

Answer C is correct because when a contingency is resolved immediately subsequent to the issuance of 30. Which of the following material events occurring subsequent to the December 31, 2005 balance
a report which was qualified with respect to the contingency, the auditor is not required to take any action sheet would not ordinarily result in an adjustment to the financial statements before they are
regarding the event. Resolution of the contingency after the issuance of the report is not subsequent issued on March 2, 2006?
discovery of facts existing at the date of the auditor's report. Because the contingency did result in a a) Write-off of a receivable from a debtor who had suffered from deteriorating financial condition
qualified report, the auditor was aware of the facts existing at the date of the auditor's report. Also, the for the past 6 years. The debtor filed for bankruptcy on January 23, 2006.
auditor's responsibility for continuing inquiry is precluded by AU 561. b) Acquisition of a subsidiary on January 23, 2006, negotiation had begun in December 2003.
c) Settlement of extended litigation on January 23, 2006, in excess of the recorded year-end
Answer D is incorrect. Refer to the correct answer explanation. liability.
d) A 3-for-5 reserve stock spit consummated on January 23, 2006.

28. Under which of the following circumstances may audited financial statements contain a note Answer A is incorrect because the debtor's deteriorating financial condition was in existence at year-end.
disclosing a subsequent event which is labeled unaudited? Therefore, an adjustment to the financial statements is appropriate.
a) When the subsequent event does not require adjustment of the financial statements.
b) When the event occurs after completion of fieldwork and before issuance of the auditor’s Answer B is correct because the condition (acquisition of a subsidiary) did not arise until after year-end.
report. Footnote disclosure of this transaction, however, is necessary.
c) When audit procedures with respect to the subsequent event were not performed by the
auditor. Answer C is incorrect because the condition giving rise to the litigation existed at the balance sheet date.
d) When the event occurs between the date of the auditor’s original report and the date of the Therefore, settlement of the litigation would require an adjustment to the financial statements.
reissuance of the report.
Answer D is incorrect because APB 15 requires retroactive adjustment for such stock splits.
Answer A is incorrect. Refer to the correct answer explanation.

Answer B is incorrect because if the auditor is aware of a subsequent event that has occurred after the 31. Which event that occurred after the end of the fiscal year under audit but prior to issuance of the
completion of fieldwork but before issuance of the report which should be disclosed, the auditor may auditor’s report would not require disclosure in the financial statements?
either dual date the report or date the report as of the date of the subsequent event and extend the a) Sale of the bond or capital stock issue.
procedures for review of subsequent events to that date. The auditor may not label the note unaudited. b) Loss of plant or inventories as result of fire or flood.
6
Audit Report
c) A major drop in the quoted market price of the stock of the corporation.
d) Settlement of litigation when the event giving rise to the claim took place after the balance Answer B is incorrect because additional procedures are required when a decision is made not to make
sheet date. reference to the other auditor.

Answer A is incorrect because AU 560 indicates that the sale of a bond issue or issuance of capital stock Answer C is correct because when the principal auditor finds it impractical to review the work of another
is a subsequent event which should be disclosed. auditor, the principal auditor will make reference to the examination of the other auditor and issue an
unqualified report. This reference will indicate the division of responsibility between that portion of the
Answer B is incorrect because AU 560 indicates that a loss of plant or inventories due to a natural financial statements covered by the principal auditor's own examination and that covered by the
catastrophe is a subsequent event which should be disclosed. examination of the other auditor.

Answer C is correct because a major drop in the quoted market price of the corporation's stock would not Answer D is incorrect because the division of responsibility within an audit report is not considered an
have financial statement effects and accordingly, need not be disclosed as a subsequent event. "except for" qualified opinion.

Answer D is incorrect because AU 560 indicates that the settlement of litigation is a subsequent event
which should be disclosed. 34. The principal auditor is satisfied with the independence and professional reputation of the other
auditor who has audited a subsidiary but wants to indicate the division of responsibility. The
principal auditor should
32. Jones, CPA, is the principal auditor who is auditing the consolidated financial statements of his a) Modify the scope paragraph of the report.
client. Jones plans to refer to another CPA’s examination of the financial statements of a b) Modify the introductory, scope, and opinion paragraphs of the report.
subsidiary company but does not wish to present the other CPA’s audit report. Both Jones and c) Not modify the report except for inclusion of an explanatory middle paragraph.
the other CPA’s audit reports have noted no exceptions to generally accepted accounting d) Modify the opinion paragraph of the report.
principles. Under these circumstances the opinion paragraph of Jones’ consolidated audit report
should express Answer A is incorrect. Refer to the correct answer explanation.
a) An unqualified opinion.
b) A disclaimer of opinion. Answer B is correct because when a principal auditor wants to indicate division of responsibility with
c) An “except for” opinion. another auditor, the principal auditor should refer to the examination of the other auditor in the
d) A principal opinion. introductory, scope, and opinion paragraphs of the auditor report.

Answer A is correct. When a principal CPA refers to another CPA's examination, the CPA's audit report Answer C is incorrect. Refer to the correct answer explanation.
is considered an unqualified opinion. The report of the other auditor is only required to be presented if the
other auditor is named (note that the other auditor does not have to be and is usually not named). Answer D is incorrect. Refer to the correct answer explanation.

Answer B is incorrect. Refer to the correct answer explanation.


35. When the report of a principal auditor makes reference to the examination made by another
Answer C is incorrect. Refer to the correct answer explanation. auditor, the other auditor may be named if express permission to do so is given and
a) The report of the principal auditor names the other auditor in both the scope and opinion
Answer D is incorrect. Refer to the correct answer explanation. paragraphs.
b) The principal auditor accepts responsibility for the work of the other auditor.
c) The report of the other auditor is presented together with the report of the principal auditor.
33. Thomas, CPA, has examined the consolidated financial statements of Kass Corporation. Jones, d) The other auditor is not an associate or correspondent firm whose work is done at the request
CPA, has examined the financial statements of the sole subsidiary which is material in relation to of the principal auditor.
the total examined by Thomas. It would be appropriate for Thomas to serve as the principal
auditor, but it is impractical for Thomas to review the work of Jones. Assuming an unqualified Answer A is incorrect because there is no requirement that the other auditor be named in both the scope
opinion is expressed by Jones, one would expect Thomas to and opinion paragraphs.
a) Refuse to express an opinion on the consolidated financial statements.
b) Express an unqualified opinion on the consolidated financial statements and not refer to the Answer B is incorrect because the principal auditor would have no need to make reference to the other
work of Jones. auditor if the principal was willing to accept responsibility for the work of the other auditor.
c) Express an unqualified opinion on the consolidated financial statements and refer to the work
of Jones. Answer C is correct because in making reference to the other auditor, they may be expressly named only
d) Express an “except for” opinion on the consolidated financial statements and refer to the work if permission has been granted and the report of the other auditor is presented along with the principal
of Jones. auditor's report.

Answer A is incorrect because if it is appropriate for Thomas to be the principal auditor, and if Jones has Answer D is incorrect because there is no differentiation between other auditors who are associates or
issued an unqualified report, there is no reason to refuse to express an opinion. correspondent firms and those who are not.
7
Audit Report
Answer C is incorrect because the entity's lack of experience about such litigation is an uncertainty that
36. Which of the following matters is an auditor required to communicate to an entity’s audit may result in an unqualified opinion with an explanatory paragraph.
committee?
a) The basis for assessing control risk below the maximum. Answer D is incorrect because the lack of insurance coverage is an aspect of the uncertainty involved
b) The process used by management in formulating sensitive accounting estimates. that may lead to an unqualified report with an explanatory paragraph.
c) The auditor’s preliminary judgments about materiality levels.
d) The justification for performing substantive procedures at interim dates.
39. Grant Company’s financial statements adequately disclose uncertainties that concern future
Answer A is incorrect. Refer to the correct answer explanation. events, the outcome of which are not susceptible to reasonable estimation. The auditor’s report
should include a(n)
Answer B is correct because one of the areas listed is the process used by management in formulating a) Unqualified opinion.
sensitive accounting estimates. b) “Subject to” qualified opinion.
c) “Except for” qualified opinion.
Answer C is incorrect. Refer to the correct answer explanation. d) Adverse opinion.

Answer D is incorrect. Refer to the correct answer explanation. Answer A is correct. Grant Company adequately disclosed the uncertainties that are not susceptible of
reasonable estimation as per GAAP. Such matters are to be regarded as uncertainties for the purposes
of considering the need for adding an explanatory paragraph to what remains an unqualified opinion.
37. The management of Stanley Corporation has decided not to account for a material transaction in
accordance with the provisions of a recent statement of the FASB. They have set forth their Answer B is incorrect. "Subject to" qualified opinions are no longer issued.
reasons in note “B” to the financial statements which clearly demonstrates that due to unusual
circumstances the financial statements would otherwise have been misleading. The auditor’s Answer C is incorrect. The proper treatment of uncertainties within an entity's financial statements will not
report will probably contain a(n) result in an "except for" qualified opinion.
a) Consistency language and reference to note “B”.
b) Unqualified opinion and the an explanatory paragraph. Answer D is incorrect. The proper treatment of uncertainties within an entity's financial statements will not
c) Disclaimer and an explanatory paragraph. result in an adverse opinion.
d) “Except for” opinion and an explanatory paragraph.

Answer A is incorrect because there is no indication that the financial statements are inconsistent with 40. When an auditor concludes there is substantial doubt about an entity’s ability to continue as a
those of the prior year. Refer to the correct answer explanation. going concern for a reasonable period of time, the auditor’s responsibility is to
a) Prepare prospective financial information to verify whether management’s plans can be
Answer B is correct because when unusual circumstances are present that would cause financial effectively implemented.
statements to be misleading if the statement of FASB was followed, an unqualified opinion and an b) Project future conditions and events for a period of time not to exceed 1 year following the
explanatory paragraph are appropriate. date of the financial statements.
c) Issue a qualified or adverse opinion, depending upon materiality, due to the possible effects
Answer C is incorrect. Refer to the correct answer explanation. on the financial statements.
d) Consider the adequacy of disclosure about the entity’s possible inability to continue going
Answer D is incorrect. Refer to the correct answer explanation. concern.

Answer A is incorrect. The auditor is not required to prepare prospective financial information.
38. Tech Company has disclosed an uncertainty due to pending litigation. The auditor’s decision to
issue a qualified opinion rather than an unqualified opinion with an explanatory paragraph most Answer B is incorrect. While an auditor must consider management's plans, he or she does not project
likely would be determined by the future conditions and events.
a) Lack of sufficient evidence.
b) Inability to estimate the amount of loss. Answer C is incorrect. Neither a qualified or an adverse opinion is necessary when the disclosures are
c) Entity’s lack of experience with such litigation. adequate.
d) Lack of insurance coverage for possible losses from such litigation.
Answer D is correct. When, after considering management's plans, the auditor concludes there is
Answer A is correct because the lack of sufficient evidence is a scope limitation that may result in a substantial doubt, he or she should consider the possible effects on the financial statements, and the
qualified opinion. adequacy of the related disclosure. In addition, an explanatory paragraph should be added to the audit
report.
Answer B is incorrect because inability to estimate the amount of loss is an uncertainty that may result in
an unqualified opinion with an explanatory paragraph.
8
Audit Report
41. Kane, CPA, concludes that there is substantial doubt about Lima Co.’s ability to continue as a Answer B is correct because a change from an unacceptable accounting principle to an acceptable one
going concern for a reasonable period of time. If Lima’s financial statements adequately results in a consistency modification.
disclose its financial difficulties, Kane’s auditor’s report is required to include an explanatory
paragraph that specifically uses the phrase(s) Answer C is incorrect because an error not involving an accounting principle (e.g., a clerical error) is
treated as a prior period adjustment with no consistency modification.
Possible discontinuance of Reasonable period of time no to
operations exceed 1 year Answer D is incorrect because changes in classification do not result in consistency modifications.
A Yes Yes
B Yes No
C No Yes 44. A material change in an accounting estimate
D No No a) Requires a consistency modification in the auditor’s report and disclosure in the financial
statements.
Answer A is incorrect. Refer to the correct answer explanation. b) Requires a consistency modification in the auditor’s report but does not require disclosure in
the financial statements.
Answer B is incorrect. Refer to the correct answer explanation. c) Affects comparability and may require disclosure in a note to the financial statements but does
not require a consistency modification in the auditor’s report.
Answer C is incorrect. Refer to the correct answer explanation. d) Involves the acceptability of the generally acceptable accounting principles used.

Answer D is correct because the explanatory paragraph presented in AU 341 includes neither of the Answer A is incorrect because a consistency modification is not necessary.
terms.
Answer B is incorrect because a consistency modification is not necessary.

42. An auditor concludes that there is substantial doubt about an entity’s ability to continue as a Answer C is correct. The requirement is to determine the auditing and reporting treatment of a material
going concern for a reasonable period of time. If the entity’s disclosures concerning this matter change in an accounting estimate. While a consistency modification is not necessary, footnote disclosure
are adequate, the audit report may include a(n) is required.

Disclaimer of opinion Except for qualified opinion Answer D is incorrect because a change in accounting principle has not occurred.
A Yes Yes
B No No
45. Which of the following requires recognition in the auditor’s opinion as to consistency?
C No Yes
a) Changing the salvage value of an asset.
D Yes No b) Changing the presentation of prepaid insurance from inclusion in “other assets” to disclosing it
as a separate item.
Answer A is incorrect. Refer to the correct answer explanation. c) Division of the consolidated subsidiary into two subsidiaries which are both consolidated.
d) Changing from consolidated a subsidiary to carrying it on the equity basis.
Answer B is incorrect. Refer to the correct answer explanation.
Answer A is incorrect because changing the salvage value of an asset is considered a change in an
Answer C is incorrect. Refer to the correct answer explanation. accounting estimate which does not affect consistency.
Answer D is correct. A disclaimer of opinion may be appropriate but an "except for" qualified opinion is Answer B is incorrect because changing the presentation of prepaid insurance from inclusion in other
never appropriate when substantial doubt about an entity's ability to continue as a going concern has assets to a separate item is a change in classification which does not affect consistency.
been adequately disclosed.
Answer C is incorrect because division of a consolidated subsidiary into two consolidated subsidiaries is a
change in classification not affecting consistency.
43. Which of the following will result in explanatory language as to consistency in the auditor’s
report, whether or not the item is fully disclosed in the financial statements? Answer D is correct. Changing from consolidating a subsidiary to carrying on the equity basis is a change
a) A change in accounting estimate. in reporting entity. Changes in reporting entities, changes in accounting principles, correction of errors,
b) A change from an unacceptable accounting principle to a generally accepted one. and changes in principles inseparable from changes in estimates all affect consistency and must be
c) Correction of an error not involving a change in accounting principle. referred to in the auditor's opinion.
d) A change in classification.

Answer A is incorrect because a change in accounting estimate requires no consistency modification. 46. With respect to consistency, which of the following should be done by an independent auditor,
who has not examined a company’s financial statements for the preceding year but is doing so
in the current year?
9
Audit Report
a) Report on the financial statements of the current year without considering consistency with the c) The auditor is consulted before material changes are made in the application of accounting
preceding year. principles.
b) Considering the consistent application of principles within the year under examination but not d) The comparability of financial statements between periods is not materially affected by
between the current and preceding year. changes in accounting principles without disclosure
c) Adopt procedures that are practicable and reasonable in the circumstances to obtain
assurance that the principles employed are consistent between the current and preceding Answer A is incorrect since there may be variations in format and presentation of financial statements.
year.
d) Rely on the report of the prior year’s auditors if such a report does not provide explanatory Answer B is incorrect since the consistency standard only requires the communication of the fact that a
languages as to consistency. lack of consistency exists in the handling of items between periods.

Answer A is incorrect because the only case where the auditor would not consider consistency is when Answer C is incorrect because there is no requirement that an auditor be consulted before material
covering the first year of client operations. Consistency must be considered in this situation because the changes are made in the application of accounting principles (although this may be done if the client so
report is not covering the client's initial period. desires).

Answer B is incorrect because the consistency standard refers to the consistent application of GAAP Answer D is correct. The requirement is to determine the objective of the consistency standard. The
between periods, not within periods. objective is (1) to give assurance that the comparability of financial statements between periods has not
been materially affected by changes in accounting principles, or (2) if comparability has been materially
Answer C is correct because during the first examination, the auditor should adopt procedures that are affected by such changes, to require appropriate reporting by the independent auditor regarding such
practicable and reasonable to assure that accounting principles are applied consistently between the changes.
current and the preceding year.

Answer D is incorrect because the prior year's audit report does not specify the accounting principles 49. The following explanatory paragraph was included in an auditor’s report to indicate a lack of
used in the prior year. In addition, explanatory language in the prior year's audit report as to consistency consistency:
would indicate an inconsistency between the prior year and its previous year, while consistency in the
current year would not be addressed. “As discussed in note T to the financial statements, the company changed its method of computing
depreciation in 200X.”

47. The auditor concludes that there is a material inconsistency in the other information in an annual How should the auditor report on this matter if the auditor concurred with the change?
report to shareholders containing audited financial statements. If the client refuses to revise or
eliminate the material inconsistency, the auditor should Type of opinion Location of explanatory paragraph
a) Revise the auditor’s report to include a separate explanatory paragraph describing the A Unqualified Before opinion paragraph
material inconsistency. B Unqualified After opinion paragraph
b) Consult with a party whose advice might influence the client, such as the client’s legal counsel. C Qualified Before opinion paragraph
c) Issue a qualified opinion after discussing the matter with the client’s board of directors. D Qualified After opinion paragraph
d) Consider the matter closed since the other information is not in the audited financial
statements. Answer A is incorrect. Refer to the correct answer explanation.
Answer A is correct because if the incorrect information is not revised to eliminate the material Answer B is correct. Inconsistency in the application of GAAP usually results in an unqualified report with
inconsistency, the auditor should consider actions such as revising the audit report to include an explanatory language. Explanatory paragraphs for inconsistency in unqualified reports follow the opinion
explanatory paragraph, withholding use of the audit report, and withdrawing from the engagement. paragraph.
Answer B is incorrect because the CPA need not consult with the client's legal counsel. Answer C is incorrect. Refer to the correct answer explanation.
Answer C is incorrect because a qualified opinion may or may not be necessary (depending upon Answer D is incorrect. Refer to the correct answer explanation.
whether the financial statements are misstated) and because the board of directors need not be
consulted.
50. The consistency standard does not apply to an accounting change that results from a change in
Answer D is incorrect because the auditor needs to consider the other information as indicated above. a) An accounting principle that is not generally accepted.
b) An accounting estimate
c) The reporting entity.
48. The objective of the consistency standard is to provide assurance that d) An accounting principles inseparable from a change in accounting estimate.
a) There are no violation in the formal and presentation of financial statements.
b) Substantially different transactions and events are not accounted for on identical basis.

10
Audit Report
Answer A is incorrect because a change from an accounting principle that is not generally accepted to 53. The prior year’s financial statements of YZ, Inc., which were audited by Pate, CPA, are
one that is generally accepted is considered a correction of an error in principle which does not affect the presented for comparative purposes without Pate’s audit report. Jennings, CPA, the successor
consistency standard. auditor, should indicate in the current year audit report that the prior year’s financial statements
were examined by another auditor
Answer B is correct because a change in accounting estimate does not affect the consistency standard. a) Only if pate’s opinion was other than unqualified.
Accordingly, this change need not be mentioned in the audit report but may require disclosure in the b) But should not indicate the type of opinion expressed by Pate.
footnotes to the financial statements. c) Only if the prior year’s financial statements have been restated.
d) But should not name Pate as the predecessor auditor.
Answer C is incorrect because a change in reporting entity is a special type of change in accounting
principle to which the consistency standard is generally applicable. Answer A is incorrect. Refer to the correct answer explanation.

Answer D is incorrect because a change in accounting principle inseparable from a change in accounting Answer B is incorrect. Refer to the correct answer explanation.
estimate does affect the consistency standard.
Answer C is incorrect. Refer to the correct answer explanation.

51. When comparative financial statements are presented, the fourth standard of reporting, which Answer D is correct because the AU 508 requires that the predecessor auditor not be named, but that the
refers to financial statements “taken as a whole,” should be considered to apply to the financial following information be indicated in the scope paragraph of the successor auditor's report: (1) the
statements of the financial statements of the prior period were examined by other auditors, (2) the date of the other auditor's
a) Periods presented plus one preceding period. report, (3) the type of opinion expressed, and (4) the substantive reasons therefore, if it was other than
b) Current period only. unqualified.
c) Current period and those of the other periods presented.
d) Current and immediately preceding period only.
54. When the financial statements of a nonpublic entity for a prior period have not been audited and
Answer A is incorrect. Refer to the correct answer explanation. are presented, for comparative purposes, with current period statements that have been audited,
a) The auditor should request removal of the unaudited statements since it is improper to present
Answer B is incorrect. Refer to the correct answer explanation. them for comparative purposes with audited statements.
b) The auditor should identify the financial statements that were not examine in a separate
Answer C is correct. The professional standards state that this term should be considered to apply not paragraph in the auditor’s report accompanying the current statements.
only to the financial statements of the current period but also to those of one or more prior periods that c) The unaudited statements do not need to be marked “unaudited” as this may confuse the
are presented on a comparative basis with those of the current period. users of the statements.
d) The auditor’s report accompanying the statements should not mention that the prior period
Answer D is incorrect. Refer to the correct answer explanation. statements are unaudited, but the unaudited statements should be marked “unaudited”.

Answer A is incorrect because the unaudited statements need not be removed.


52. When financial statements of a prior period are presented on a comparative basis with financial
statements of the current period, the continuing auditor is responsible for Answer B is correct. The requirement is to determine the auditor's reporting responsibility when
a) Expressing dual date opinions. comparative statements for a nonpublic entity are being issued and the prior period has not been audited.
b) Updating the report on the previous financial statements only if there has not been a change in In such circumstances the auditor may add a separate paragraph to the audit report and identify the
opinion. financial statements.
c) Updating the report on the previous financial statements only if the previous report was
qualified and the reasons for qualification no longer exist. Answer C is incorrect because the unaudited statements must be marked "unaudited."
d) Updating the report on the previous financial statements regardless of the opinion previously
issued. Answer D is incorrect because the auditor's report should mention that the prior statements are
unaudited.
Answer A is incorrect. Refer to the correct answer explanation.

Answer B is incorrect. Refer to the correct answer explanation. 55. Comparative financial statements include the financial statements of a prior period which were
examined by a predecessor auditor, whose report is not presented. If the predecessor auditor’s
Answer C is incorrect. Refer to the correct answer explanation. report was qualified, the successor auditor must
a) Express an opinion on the current year statements alone and make no reference to the prior
Answer D is correct because a continuing auditor must update his report on the financial statements of year statements.
the one or more prior periods presented regardless of the opinion previously issued. b) Issue a standard short-form comparative report indicating the division of responsibility.
c) Obtain written approval from the predecessor auditor to include the prior year’s financial
statements.
11
Audit Report
d) Disclose the reasons for any qualification included in the predecessor auditor’s opinion. Answer D is correct because when a predecessor auditor reissues an audit report, without revising the
original wording, the auditor should use the date of the previous report. This avoids any implications that
Answer A is incorrect. Refer to the correct answer explanation. the auditor has re-examined any records, transactions, or events after the date of the original report.

Answer B is incorrect because division of responsibility in an audit report applies only when a portion of 1
year's examination has been performed by other auditors. 58. An auditor has previously expressed a qualified opinion on the financial statements of a prior
period because of a departure from generally accepted accounting principles. The prior-period
Answer C is incorrect because written approval from the predecessor need not be obtained in order to financial statements are restated in the current period to conform with generally accepted
include the prior year's financial statements. accounting principles. The auditor’s updated report on the prior-period financial statements
should
Answer D is correct because AU 508 specifies when the predecessor auditor's opinion was other than a) Express an unqualified opinion concerning the restated financial statements.
unqualified, the successor must describe the nature of any reasons for the qualification. b) Be accompanied by the original auditor’s report on the prior period.
c) Bear the same date as the original auditor’s report on the prior period.
d) Qualify the opinion concerning the restated financial statements because of a change in
56. When a predecessor auditor reissues the report on the prior period’s financial statements at the accounting principle.
request of the former client, the predecessor auditor should
a) Indicate in the introductory paragraph of the reissued report that the financial statements of the Answer A is correct because the auditor should indicate that the statements have been restated and
subsequent period were audited by another CPA. should express an unqualified opinion with respect to the restated financial statements. An explanatory
b) Obtain an updated management representation letter and compare it to that obtained during paragraph to the report should disclose (1) the date of the auditor's previous report, (2) the type of opinion
the prior period audit. previously expressed, (3) the circumstances or events that caused the auditor to express a different
c) Compare the prior period’s financial statements that the predecessor reported on with the opinion, and (4) that the updated opinion differs from the previous opinion.
financial statements to be presented for comparative purposes.
d) Add an explanatory paragraph to the reissued report stating that the predecessor has not Answer B is incorrect because the original auditor's report has been modified.
performed additional auditing procedures concerning the prior period’s financial statements.
Answer C is incorrect because the date of the comparative statement audit report is the last day of
Answer A is incorrect because the predecessor auditor should not indicate in the introductory paragraph fieldwork for the current year audit.
of the reissued report that the financial statements of the subsequent period were audited by another
CPA. Answer D is incorrect because a change in accounting principle results in an unqualified opinion with an
explanatory paragraph, not a qualified opinion.
Answer B is incorrect because no updated management representation letter need be obtained.

Answer C is correct because the predecessor auditor should (1) read the financial statements of the 59. Before reissuing a report which was previously issued on the financial statements of a prior
current period, (2) compare the prior period financial statements with the subsequent financial statements period, a predecessor auditor should
being presented for comparative purposes, and (3) obtain a letter of representations from the successor a) Review the successor auditor’s working papers.
auditor. b) Examine significant transactions or events since the date of previous issuance.
c) Obtain a signed engagement letter from the client.
Answer D is incorrect because no explanatory paragraph should be added to the reissued report. See d) Obtain a letter of representation from the successor auditor.
AU 508 for predecessor responsibilities with respect to comparative financial statements.
Answer A is incorrect since the various Statements on Auditing Standards contain no requirement that the
predecessor auditor review the successor auditor's working papers.
57. After performing all necessary procedures a predecessor auditor reissues a prior-period report
on financial statements at the request of the client without revising the original wording. The Answer B is incorrect. Refer to the correct answer explanation.
predecessor auditor should
a) Delete the date of the report. Answer C is incorrect. Refer to the correct answer explanation.
b) Dual date the report.
c) Use the reissue date. Answer D is correct because AU 508 states that a predecessor auditor should obtain a letter of
d) Use the date of the previous report. representation from the successor auditor. The predecessor auditor must also read the current financial
statements and compare them to the prior-period statements.
Answer A is incorrect. Refer to the correct answer explanation.

Answer B is incorrect. Refer to the correct answer explanation. 60. The predecessor auditor, who is satisfied after properly communicating with the successor
auditor, has reissued a report because the audit client desires comparative financial statements.
Answer C is incorrect. Refer to the correct answer explanation. The predecessor auditor’s report should make
a) No reference to the report or the work of the successor auditor.
12
Audit Report
b) Reference to the work of the successor auditor in the scope and opinion paragraphs. 63. If the auditor believes that required disclosures of a significant nature are omitted from the
c) Reference to both the work and the report of the successor auditor only in the opinion financial statements under examination, the auditor should decide between issuing
paragraph. a) A qualified opinion or an adverse opinion.
d) Reference to the report of the successor auditor only in the scope paragraph. b) A disclaimer of opinion or a qualified opinion.
c) An adverse opinion or a disclaimer of opinion.
Answer A is correct because no reference to the work or the report of the successor auditor is to be made d) An unqualified opinion or a qualified opinion.
in this situation.
Answer A is correct because material disclosures required by GAAP, if omitted, cause the financial
Answer B is incorrect. Refer to the correct answer explanation. statements to be in violation of GAAP. When the financial statements are materially affected by a
departure from GAAP, the auditor should express a qualified or an adverse opinion.
Answer C is incorrect. Refer to the correct answer explanation.
Answer B is incorrect. Refer to the correct answer explanation.
Answer D is incorrect. Refer to the correct answer explanation.
Answer C is incorrect. Refer to the correct answer explanation.

61. Which of the following is considered “unaudited” information when included with historical Answer D is incorrect. Refer to the correct answer explanation.
financial statement?
a) Interim information.
b) Segmental information. 64. Which of the following narrative disclosures appearing in notes to financial statements would an
c) Notes to the financial statements. auditor be most likely to consider inappropriate?
d) Investment security classifications. a) The related-party transaction was consummated on terms no less favorable than those that
would have been obtained if the transaction had been with an unrelated party.
Answer A is correct because that information should be marked "unaudited." b) The accounts of subsidiaries in which the corporation has more than 50% ownership are fully
consolidated.
Answer B is incorrect because segment information is required by SFAS 14 to be disclosed. c) Legal and other costs associated with the covenant-not-to-compete will be amortized using the
straight-line method during the next 3 years.
Answer C is incorrect because the notes are considered audited. d) Minor fluctuations in foreign currency exchange rates are reflected in the accompanying
financial statements.
Answer D is incorrect because the auditor must audit investment security classifications to obtain the
valuation objective. Answer A is correct because representations to the effect that a transaction was consummated on terms
no less favorable than those that would have been obtained if the transaction had been with an unrelated
party are difficult to substantiate and the auditor may be unable to reach a conclusion as to the propriety
62. When a client declines to make essential disclosures in the financial statements or in the notes, thereof.
the independent auditor should
a) Provide the necessary disclosures in the auditor’s report and appropriately modify the opinion. Answer B is incorrect because it represents an acceptable practice under generally accepted accounting
b) Explain to the client that an adverse opinion must be issued. principles. Refer to the correct answer explanation.
c) Issue an unqualified report and inform the stockholders of the improper disclosure in an
“unaudited” note. Answer C is incorrect because it represents an acceptable practice under generally accepted accounting
d) Issue a disclaimer of opinion. principles. Refer to the correct answer explanation.

Answer A is correct because when a client declines to make essential disclosures, the independent Answer D is incorrect because minor fluctuations in foreign currency rates need not be reflected in
auditor should provide the disclosures where practicable, and modify the audit opinion. financial statements. Refer to the correct answer explanation.

Answer B is incorrect because a qualified opinion may be appropriate rather than an adverse opinion.
Therefore, an adverse opinion is not always required in this situation. 65. If information accompanying the basic financial statements in an auditor-submitted document
has been subjected to auditing procedures, the auditor may express an opinion which states that
Answer C is incorrect because improper disclosures result in a qualified or adverse audit opinion. An the accompanying information is fairly stated in
unaudited note would not be appropriate in view of the audit. a) Conformity with generally accepted accounting principles.
b) Terms of negative assurance.
Answer D is incorrect because a disclaimer of opinion is not appropriate when there are departures from c) All material respects in relation to the basic financial statements taken as a whole.
GAAP, including inadequate disclosures. d) Conformity with principles for presenting accompanying information.

Answer A is incorrect because the financial statements themselves, not the information accompanying
the statements, are to follow generally accepted accounting principles.
13
Audit Report
a) An explanatory paragraph to the auditor’s report.
Answer B is incorrect because AU 551 requires either an opinion or a disclaimer of opinion on the b) An explanatory note to the financial statements.
accompanying information; negative assurance is not appropriate. c) The body of the financial statements.
d) The “summary of significant accounting policies” section of the financial statements.
Answer C is correct. The requirement is to determine an auditor's reporting responsibility for audited
information accompanying the basic financial statements in an auditor-submitted document. The report Answer A is correct because when the auditor wishes to emphasize such a matter, it should be done in
should include either an opinion or disclaimer of opinion on whether the accompanying information is an explanatory paragraph to the auditor's report. The report is still considered unqualified.
fairly stated in all material respects in relation to the basic financial statements taken as a whole.
Answer B is incorrect because notes are portions of the financial statements in which the client reports,
Answer D is also incorrect because, as indicated above, the opinion on whether the accompanying not the auditor.
information is fairly stated is with respect to the financial statements, not with respect to principles for the
accompanying information. Answer C is incorrect because the client, not the auditor, reports in the body of the financial statements.

Answer D is incorrect because the "Summary of significant accounting policies" section is a portion of the
66. Which of the following best describes the auditor’s responsibility for “other information” included financial statements in which the client reports, not the auditor.
in the annual report to stockholders which contains financial statements and the auditor’s report?
a) The auditor has no obligation to read the “other information”.
b) The auditor has no obligation to corroborate the “other information”, but should read the “other 69. An auditor includes an explanatory paragraph in an otherwise unqualified report in order to
information “ to determine whether it is materially inconsistent with the financial statements. emphasize that the entity being reported upon is a subsidiary of another business enterprise.
c) The auditor should extend the examination to the extent necessary to verify the “other The inclusion of this explanatory paragraph
information”. a) Is appropriate and would not negate to unqualified opinion.
d) The auditor must modify the auditor’s report to state that the “other information is unaudited” or b) Is considered a qualification of the report.
no covered by the auditor’s report”. c) Is a violation of generally accepted reporting standards if this information is disclosed in
footnotes to the financial statements.
Answer A is incorrect because the auditor is required to read the other information. d) Necessitates a revision of the opinion paragraph to include the phrase “with the foregoing
explanation”.
Answer B is correct because AU 550 requires that an auditor read the "other information" to determine
whether it is inconsistent with the financial statements. Answer A is correct because the report is still considered unqualified.

Answer C is incorrect because the auditor is not required to extend the examination to verify the "other Answer B is incorrect because the report is still considered unqualified.
information."
Answer C is incorrect because footnote disclosure of such an emphasis of a matter is allowed.
Answer D is incorrect because the auditor's report is not to be modified in such circumstances.
Answer D is incorrect because the words "with the foregoing explanation" should not be added to the
opinion paragraph.
67. It is not appropriate to refer a reader of an auditor’s report to a financial statement note for
details concerning
a) Subsequent events. 70. A company issues audited financial statements under circumstances which require the
b) The pro forma effects of a business combination. presentation of a statement of cash flows. If the company refuses to present a statement of
c) Sales of a discontinued operation. cash flows, the independent auditor should
d) The results of confirmation of receivables. a) Disclaim an opinion.
b) Prepare a statement of cash flows and note in an explanatory paragraph of the report that this
Answer A is incorrect. Refer to the correct answer explanation. statement is auditor-prepared.
c) Prepare a statement of cash flows and disclose in a footnote that this statement is auditor-
Answer B is incorrect. Refer to the correct answer explanation. prepared.
d) Qualify his opinion with an “except for” qualification and a description of the omission in an
Answer C is incorrect. Refer to the correct answer explanation. explanatory paragraph of the report.

Answer D is correct because the results of confirmation of receivables (as well as the results of other Answer A is incorrect. A disclaimer of opinion is not necessary.
audit procedures) are not disclosed in a note to the financial statements.
Answer B is incorrect. The auditor is not required to prepare a statement of cash flows or any of the basic
financial statements.
68. The auditor who wishes to point out that the entity has sufficient transactions with related parties
should disclose this fact in
14
Audit Report
Answer C is incorrect. The auditor is not required to prepare a statement of cash flows or any of the 73. Under which of the following circumstances would a disclaimer of opinion not be appropriate?
basic financial statements. a) The financial statements fail to contain adequate disclosure of related-party transactions.
b) The client refuses to permit its attorney to furnish information requested in a letter of audit
Answer D is correct. Omission of a statement of cash flows results in an "except for" qualified report with inquiry.
a middle paragraph describing the omission. c) The auditor is engaged after fiscal year-end and is unable to observe physical inventories or
apply alternative procedures to verify their balances.
d) The auditor is unable to determine the amounts associated with illegal acts committed by the
71. In which of the following circumstances would an auditor be required to issue a qualified report client’s management.
with a separate explanatory paragraph?
a) The auditor satisfactorily performed alternative accounts receivable procedures because Answer A is correct. The requirement is to identify the circumstance under which a disclaimer of opinion
scope limitations prevented performance of normal procedures. would not be appropriate. A failure to contain adequate disclosure of related party transactions
b) The financial statements reflect the immaterial effects of a change in accounting principles represents a departure from generally accepted accounting principles and disclaimers are not appropriate
from one period to the next. in such circumstances.
c) A particular note to the financial statements discloses a company accounting method which
deviates from generally accepted accounting principles. Answer B is incorrect. Client refusal to permit its attorney to furnish information represents a scope
d) The financial statements of a significant subsidiary were examined by another auditor, and limitation that may lead to a disclaimer of opinion.
reference to the other auditor’s report is to be made in the principal auditor’s report.
Answer C is incorrect. The inability to observe physical inventories or to apply alternative procedures
Answer A is incorrect because a qualified opinion is not required if the auditor conducts the examination represents scope limitations that may lead to a disclaimer of opinion.
per GAAS and applies all procedures necessary in the circumstances. Here the auditor has satisfied
himself with alternative procedures. Answer D is incorrect. The inability to determine amounts of illegal acts represents a scope limitation that
may lead to a disclaimer of opinion.
Answer B is incorrect because no explanatory paragraph is required when the opinion paragraph is
modified due to an immaterial change in accounting principle.
74. An auditor’s examination reveals a misstatement in segment information that is material in
Answer C is correct because when the financial statements are affected by a departure from generally relation to the financial statements taken as a whole. If the client refuses to make modifications
accepted accounting principles, the auditor will express a qualified opinion. When the auditor intends to to the presentation of segment information, the auditor should issue a(n)
express a qualified report, s/he should disclose all the substantive reasons in a separate explanatory a) “Except for” opinion.
paragraph. b) “Subject to” opinion.
c) Unqualified opinion.
Answer D is incorrect because a reference to another auditor's report does not constitute a qualification of d) Disclaimer of opinion.
the auditor's report.
Answer A is correct because this, like other departures from GAAP, leads to either a qualified "except for"
opinion or an "adverse" opinion. Note that an adverse opinion is not listed as a choice.
72. A CPA engaged to examine financial statements observes that the accounting for a certain
material item is not in conformity with generally accepted accounting principles, and that this fact Answer B is incorrect. "Subject to" opinions are no longer issued.
is prominently disclosed in a footnote to the financial statements. The CPA should
a) Express an unqualified opinion and insert an explanatory paragraph emphasizing the matter Answer C is incorrect. Refer to the correct answer explanation.
by reference to the footnote.
b) Disclaim an opinion. Answer D is incorrect. Refer to the correct answer explanation.
c) Not allow the accounting treatment for this item to affect the type of opinion because the
deviation from generally accepted accounting principles was disclosed.
d) Qualify the opinion because of the deviation from generally accepted accounting principles. 75. When reporting on financial statements that are required to include segment information, the
auditor’s quantitative measurement of materiality with respect to segment information should be
Answer A is incorrect. Refer to the correct answer explanation. a) Disclosed.
b) Primarily related to the segment information alone.
Answer B is incorrect. Refer to the correct answer explanation. c) Primarily related to the financial statements taken as a whole.
d) Disregarded.
Answer C is incorrect. Refer to the correct answer explanation.
Answer A is incorrect because a quantitative measurement of materiality is not disclosed with financial
Answer D is correct because when financial statements are materially affected by a departure from statements.
generally accepted accounting principles an auditor should issue either a qualified or an adverse opinion.
Answer B is incorrect. Refer to the correct answer explanation.

15
Audit Report
Answer C is correct because AU 435 requires that the materiality of segment information is evaluated a) Withdrawing from the engagement.
primarily by relating the dollar magnitude of the information to the financial statements taken as a whole. b) Issuing a disclaimer of opinion.
c) Failure to uncover the illegal activities during prior audits.
Answer D is incorrect because materiality may not be disregarded. d) Reporting these activities to the audit committee.

Answer A is incorrect because withdrawing from the engagement is appropriate in certain instances such
76. When auditing a public entity’s financial statements that include segment information, the auditor as when the client refuses to accept the issued audit report and when the client continues the illegal act.
should
a) Make a certain the segment information is labeled unaudited and determine that the Answer B is correct because in such situations in which the auditor knows of the illegal act, a simple
information is consistent with audited information. disclaimer will not suffice, since the lack of disclosure represents a departure from GAAP. This departure
b) Make a certain the segment information is labeled unaudited and perform only analytical from GAAP will result in a qualified opinion or an adverse opinion.
procedures on the segment information.
c) Audit the segment information, and, if the information is adequate and in conformity with Answer C is incorrect because an audit cannot be expected to provide assurance that illegal acts will be
GAAP, do not make reference to the segment information in the auditor’s report. detected.
d) Audit the segment information and, if the information is adequate and in conformity with
GAAP, refer to the segment information in the auditor’s report. Answer D is incorrect because the board of directors and/or its audit committee may need to be notified of
such situations by the auditor.
Answer A is incorrect because segment information is audited, not unaudited. Refer to the correct
answer explanation.
79. Because of the pervasive effects of laws and regulations on the financial statements of
Answer B is incorrect because segment information is audited, not unaudited. Refer to the correct governmental units, an auditor should obtain written management representations
answer explanation. acknowledging that management has
a) Identified and disclosed all laws and regulations that have a direct material effect on its
Answer C is correct because no reference need be made to the segment information when the financial statements.
information has been audited and is found to be adequate and in conformity with GAAP. b) Implemented internal control policies and procedures designed to detect all illegal acts.
c) Expressed both positive and negative assurance to the auditor that the entity complied with all
Answer D is incorrect. Refer to the correct answer explanation. laws and regulations.
d) Employed internal auditor who can report their findings, opinion, and conclusions objectively
without fear of political repercussion.
77. An auditor’s report included an additional paragraph disclosing that there is a difference of
opinion between the auditor and the client for which the auditor believed an adjustment to the Answer A is correct because, in addition to normal representations, auditors should consider obtaining
financial statements should be made. The opinion paragraph of the auditor’s report most likely additional representations from management acknowledging that (1) management is responsible for the
expressed a(n) entity's compliance with laws and regulations and (2) management has identified and disclosed to the
a) Unqualified opinion. auditor all laws and regulations that have a direct and material effect on the financial statements.
b) “Except for” opinion.
c) “Subject to” opinion. Answer B is incorrect. Refer to the correct answer explanation.
d) Disclaimer of opinion.
Answer C is incorrect. Refer to the correct answer explanation.
Answer A is incorrect because, if a difference of opinion is important enough to note, an unqualified
opinion would not be appropriate. Answer D is incorrect. Refer to the correct answer explanation.

Answer B is correct because a difference of opinion between the auditor and the client will generally
originate from a departure from GAAP. Therefore, a qualified "except for" opinion or an adverse opinion 80. When the financial statements contain a departure from generally accepted accounting
would be appropriate. principles, the effect of which is material, the auditor should
a) Qualify the opinion and explain the effect of the departure from generally accepted accounting
Answer C is incorrect because "subject to" opinions are no longer permissible. principles in a separate paragraph.
b) Qualify the opinion and describe the departure from generally accepted accounting principles
Answer D is incorrect because a disclaimer of opinion is not appropriate when a difference of opinion within the opinion paragraph.
exists between the auditor and client. c) Disclaim an opinion and explain the effect of the departure from generally accepted
accounting principles in a separate paragraph.
d) Disclaim an opinion and describe the departure from generally accepted accounting principles
78. When management refuses to disclose illegal activities which were identified by the independent within the opinion paragraph.
auditor, the independent auditor may be charged with violating the CPA Code of Professional
Conduct for
16
Audit Report
Answer A is correct because per AU 508 an auditor must qualify the opinion paragraph and explain the magnitude of the scope limitation described in this question. The auditor could not provide the necessary
effect of the departure from GAAP in a separate paragraph (or issue an adverse opinion). reasonable basis for an unqualified opinion.

Answer B is incorrect because the description of the departure from GAAP is in a separate paragraph, not Answer D is incorrect. An unqualified opinion is expressed by an auditor when his audit has provided a
in the opinion paragraph. reasonable basis for the opinion that the financial statements are fairly presented in accordance with
GAAP. Such an opinion cannot be expressed (with or without modification of the scope paragraph)
Answer C is incorrect because an auditor issues a disclaimer when the auditor has not performed an because of the magnitude of the scope limitation described in this question. The auditor could not
examination sufficient in scope to enable him to form an opinion. A disclaimer should not be expressed provide the necessary reasonable basis for an unqualified opinion.
because the auditor believes, based on his examination, that there are material departures from GAAP.

Answer D is incorrect both because the auditor cannot disclaim an opinion for a material departure from 83. Which of the following will not result in modification of the auditor’s report due to a scope
GAAP and because a separate paragraph is used in the audit report. limitation?
a) Restrictions imposed by the client.
b) Reliance placed on the report of another auditor.
81. An auditor’s report that refers to the use of an accounting principle at variance with generally c) Inability to obtain sufficient competent evidential matter.
accepted accounting principles contains the words, “In our opinion, with the foregoing d) Inadequacy in the accounting records.
explanation, the financial statements referred to above present fairly...” This is considered an
a) Adverse opinion. Answer A is incorrect because restrictions imposed by the client are considered scope limitations.
b) “Except for” qualified opinion.
c) Unqualified opinion with an explanatory paragraph. Answer B is correct because reliance on the report of another auditor does not constitute a qualification of
d) Example of inappropriate reporting. the auditor's opinion.

Answer A is incorrect. The statement is not considered adequate. Answer C is incorrect because the inability to obtain sufficient competent evidential matter is the basis for
a scope limitation.
Answer B is incorrect. The statement is not considered adequate.
Answer D is incorrect because inadequate accounting records are considered a scope limitation imposed
Answer C is incorrect because an unqualified opinion with a separate explanatory paragraph includes no by circumstance.
modification of the opinion paragraph.

Answer D is correct. The requirement is to determine the correct response with respect to an audit report 84. Stone was asked to perform the first audit of a wholesale business that does not maintain
which describes a departure from generally accepted accounting principles with "In our opinion, with the perpetual inventory records. Stone has observed the current inventory but has not observed the
foregoing explanation, the financial statements referred to above present fairly..." AU 508 states that physical inventory at the previous year-end date and concludes that the opening inventory
such a statement is not clear or forceful enough and therefore should not be used. balance, which is not auditable, is a material factor in the determination of cost of goods sold for
the current year. Stone will probably
a) Decline the engagement.
82. Restrictions imposed by a client prohibit the observation of physical inventories, which account b) Express an unqualified opinion on the balance sheet and income statement except for
for 35% of all assets. Alternative audit procedures cannot be applied, although the auditor was inventory.
able to examine satisfactory evidence for all other items in the financial statements. The auditor c) Express an unqualified opinion on the balance sheet and disclaim an opinion on the income
should issue a(n) statement.
a) “Except for” qualified opinion. d) Disclaim an opinion on the balance sheet and income statement.
b) Disclaimer of opinion.
c) Unqualified opinion with a separate explanatory paragraph. Answer A is incorrect because no reason is given for declining the engagement. The inability to verify the
d) Unqualified opinion with an explanation in the scope paragraph. opening inventory balance will affect the audit but is not a reason to decline the engagement.

Answer A is incorrect. A qualified opinion states that, except for the effects of the matter(s) to which the Answer B is incorrect because it describes a form of audit report not permitted under GAAS.
qualification relates, the financial statements are fairly presented in conformity with GAAP. Due to the
significant nature of this limitation, a qualified report is not appropriate in this situation. Answer C is correct because for the case described in this question, the auditor will be able to gather
evidence on all year-end balances. However, evidence with respect to the beginning inventory is lacking
Answer B is correct. AU 508 states that when restrictions that significantly limit the scope of the audit are making the verification of cost of goods sold, an income statement element, impossible. If no other
imposed by the client, ordinarily the auditor should disclaim an opinion on the financial statements. problems arise, the auditor will be able to issue an unqualified opinion on the balance sheet and a
disclaimer on the income statement.
Answer C is incorrect. An unqualified opinion is expressed by an auditor when his audit has provided a
reasonable basis for the opinion that the financial statements are fairly presented in accordance with Answer D is incorrect because the auditor is able to gather sufficient evidence on all year-end balances
GAAP. Such an opinion cannot be expressed (with or without an explanatory paragraph) because of the and, therefore, may render an opinion on the year-end balance sheet.
17
Audit Report
c) Adverse opinion.
d) Unqualified opinion with a separate explanatory paragraph.
85. A limitation on the scope of the auditor’s examination sufficient to preclude an unqualified
opinion will always result when management Answer A is incorrect because a "subject to" qualification relates to future uncertainty facing the firm and
a) Engages an auditor after the year-end physical inventory count. is therefore not appropriate.
b) Refuses to furnish a representation letter.
c) Knows that direct confirmation of accounts receivable with debtors is not feasible. Answer B is correct because in such circumstances, the auditor is to either qualify ("except for") or
d) Engages an auditor to examine only the balance sheet. disclaim an opinion, and in this question the qualify ("except for") type opinion is not one of the answer
choices.
Answer A is incorrect because AU 310 indicates that a CPA may accept an audit engagement after year-
end and still issue an unqualified opinion. In this situation, another physical inventory could be taken Answer C is incorrect because an adverse opinion is only appropriate when the auditor knows that a
which the auditor can observe, or the audit limitation could be remedied in some other manner. departure from generally accepted accounting principles exists.

Answer B is correct because when management refuses to furnish a representation letter, the scope of Answer D is incorrect because the inability to determine amounts makes an unqualified opinion
the auditor's examination has been limited sufficiently to preclude an unqualified opinion. inappropriate.

Answer C is incorrect because if direct confirmation of accounts receivable is not feasible, alternative
procedures should be employed to obtain adequate evidence necessary to satisfy the auditor as to the 88. Morris, CPA, suspects that a pervasive scheme of illegal bribes exists throughout the operations
receivables. The availability of alternative procedures to obtain adequate evidence determines whether a of Worldwide Import-Export, Inc., a new audit client. Morris notified the audit committee and
limitation of scope exists. Therefore, the inability to directly confirm receivables may or may not result in Worldwide’s legal counsel, but neither could assist Morris in determining whether the amounts
a scope limitation which precludes an unqualified opinion. involved were material to the financial statements or whether senior management was involved
in the scheme. Under these circumstances, Morris should
Answer D is incorrect because an engagement where the auditor reports only on the balance sheet is a a) Express an unqualified opinion with separate explanatory paragraph.
limitation of reporting objectives rather than a limitation of scope, as long as there is no restriction on the b) Disclaim an opinion on the financial statements.
auditor's access to information. c) Express an adverse opinion on the financial statements.
d) Issue a special report regarding the illegal bribes.

86. The auditor is unable to reach a conclusion as to the propriety of management’s Answer A is incorrect because the auditor is uncertain as to whether misstatements are involved, and
representations. The auditor will have to consider issuing a(n) therefore an unqualified opinion may not be appropriate.
a) Opinion qualified because of uncertainty.
b) Opinion qualified because of inadequate disclosure. Answer B is correct because the inability to obtain sufficient competent evidential matter will lead to a
c) Adverse opinion or qualified opinion. situation in which the auditor generally should disclaim an opinion on the financial statements.
d) Qualified opinion or a disclaimer of opinion.
Answer C is incorrect since the misstatements may not be material and therefore, an adverse opinion
Answer A is incorrect. While partially correct, it is less complete because uncertainty, as used in the may be inappropriate.
standards, only relates to estimates of events' outcomes and not to other management representations
which the auditor may question. Answer D is incorrect because special reports do not relate to illegal bribes.

Answer B is incorrect because the auditor is unlikely to know whether adequate disclosure exists in such
circumstances due to his/her overall uncertainty. 89. When a client will not make essential corporate minutes available to the auditor, the audit report
will probably contain a(n)
Answer C is incorrect because an adverse opinion is not appropriate when the auditor has been unable to a) Unqualified opinion.
resolve his/her uncertainty. b) Adverse opinion.
c) Qualified opinion.
Answer D is correct. The requirement is to determine the effect on an audit report of an auditor's inability d) Disclaimer of opinion.
to reach conclusions as to the propriety of management's representations. The auditor's uncertainty will
result in either a qualified opinion or a disclaimer of opinion based on the importance of the Answer A is incorrect because it is clearly indicated that the minutes are essential. An auditor would not
representations. issue an unqualified opinion when the client does not permit essential information to be reviewed.

Answer B is incorrect because the auditor issues an adverse opinion when the financial statements are
87. When the auditor is unable to determine the amounts associated with the illegal acts of client not presented fairly in conformity with generally accepted accounting principles. In this case, the auditor
personnel because of an inability to obtain adequate evidence, the auditor should issue a(n) does not know whether there is a departure from generally accepted accounting principles.
a) “Subject to” qualified opinion.
b) Disclaimer of opinion.
18
Audit Report
Answer C is incorrect because significant client-imposed scope restrictions will not lead to a qualified b) “Except for” qualified opinion.
opinion. c) “Subject to” qualified opinion.
d) Unqualified opinion with a separate explanatory paragraph.
Answer D is correct because when restrictions that significantly limit the scope of the audit are imposed
by the client, the auditor generally should disclaim an opinion on the financial statements. Answer A is correct because this constitutes a client imposed scope limitation which ordinarily results in a
disclaimer of opinion.

90. The auditor would most likely issue a disclaimer of opinion because of Answer B is incorrect because a disclaimer of opinion, not a qualified opinion, is generally required.
a) The client’s failure to present supplementary information require by the PAS.
b) Inadequate disclosure of material information. Answer C is incorrect because "subject to" qualified opinions are not considered appropriate under any
c) Client-impose scope limitation. circumstances.
d) The qualification of an opinion by the other auditor of a subsidiary where there is a division of
responsibility. Answer D is incorrect because a client's refusal to allow inquiry of outside legal counsel is considered to
be a scope limitation which requires a disclaimer of opinion.
Answer A is incorrect because failure to present supplementary information does not affect the auditor's
opinion, but does result in inclusion of an explanatory paragraph to the report.
93. When an independent CPA is associated with the financial statements of a publicly held entity,
Answer B is incorrect because inadequate disclosure of material information will result in qualified opinion but has not audited or reviewed such statements, the appropriate form of report to be issued
or an adverse opinion. must include a(n)
a) Negative assurance.
Answer C is correct because auditors are generally required to disclaim an opinion for client imposed b) Compilation opinion.
scope limitations. c) Disclaimer of opinion.
d) Explanatory paragraph.
Answer D is incorrect because, depending upon the circumstances involved, a qualification in the other
auditor's report may or may not lead to overall report modification. Answer A is incorrect because negative assurance requires that the auditor has performed limited
procedures in regards to the financial information. Refer to the correct answer explanation.

91. Which of the following circumstances would not be considered a departure from the auditor’s Answer B is incorrect because compilations are limited to nonpublic firms. Refer to the correct answer
standard report? explanation.
a) The auditor wishes to emphasize a particular matter regarding the financial statements.
b) The auditor’s opinion is based in part on the report of another auditor. Answer C is correct because when an independent CPA is associated with the financial statements of a
c) The financial statements are affected by a departure from a generally accepted accounting public entity that s/he has not audited or reviewed, a disclaimer of opinion should be issued. The
principle. accountant who issues this disclaimer has no responsibility to apply any procedures beyond reading the
d) The auditor is asked to report only on the balance sheet but has unlimited access to financial statements for obvious material errors.
information underlying all the basic financial statements.
Answer D is incorrect because no explanatory paragraph will be added. Refer to the correct answer
Answer A is incorrect because emphasis of a matter in the audit report constitutes a departure from the explanation.
standard report.

Answer B is incorrect because basing (in part) the auditor's opinion on the report of another auditor 94. When an adverse opinion is expressed, the opinion paragraph should include a direct reference
constitutes a departure from the standard report. to
a) A footnote to the financial statements which discusses the basis for the opinion.
Answer C is incorrect because the effect of a departure from GAAP which materially affects the financial b) The scope paragraph which discusses the basic for the opinion rendered.
statements results in a departure from the standard report. c) A separate paragraph which discusses the basis for the opinion rendered.
d) The consistency or lack of consistency in the application of generally accepted accounting
Answer D is correct because reporting on only the balance sheet is not considered a scope limitation principles.
(requiring departure from the standard report), but rather involves limited reporting objectives. The
auditor has ready access to all information underlying the financial statements and may apply all Answer A is incorrect because the auditor (not the client) should disclose all substantive reasons for the
procedures deemed necessary. adverse opinion and the principal effects on the financial position and results of operations.

Answer B is incorrect because no mention of the nonconformity with GAAP is made in the scope
92. When a client will not permit inquiry of outside legal counsel, the audit report will ordinarily paragraph.
contain a(n)
a) Disclaimer of opinion.
19
Audit Report
Answer C is correct because the opinion paragraph of an adverse opinion should refer to a separate b) Based on prospective financial information.
paragraph which discusses the basis for the opinion rendered. c) Misleading.
d) Affected by a material uncertainly.
Answer D is incorrect because adverse opinions are issued only when the financial statements taken as a
whole are not presented fairly per GAAP. Generally there is no need for reference to consistency, unless Answer A is incorrect. Inconsistency is most likely to result in an unqualified report with an additional
the auditor has specific exceptions as to consistency. explanatory paragraph. Such a report would not modify the language of the opinion paragraph.

Answer B is incorrect. If an accountant disagrees with the presentation of financial statements based on
95. An auditor would issue an adverse opinion if prospective financial information, his report includes an adverse opinion that states such financial
a) The audit was begun by other independent auditors who withdrew from the engagement. statements are not presented in conformity with guidelines established by the AICPA.
b) A qualified opinion cannot be given because the auditor lacks independence.
c) The restriction on the scope of the audit was significant. Answer C is correct. Language such as that quoted in this question is used in an adverse opinion. Such
d) The statements taken as a whole do not fairly present the financial condition and results of an opinion is appropriate when financial statements are considered to be misleading.
operations of the company.
Answer D is incorrect. Material uncertainties result in either an unqualified opinion with an explanatory
Answer A is incorrect because an auditor's report is not affected by the fact that the engagement was paragraph or a disclaimer of opinion. Neither of these types of opinions modify the opinion paragraph in
begun by other auditors who withdrew. the manner described in the question.

Answer B is incorrect because a disclaimer of opinion is issued when an auditor lacks independence.
98. In which of the following circumstances would an auditor be most likely to express an adverse
Answer C is incorrect because a scope restriction will lead to either a qualified opinion or a disclaimer of opinion?
opinion. a) The statements are not in conformity with the FASB statements regarding the capitalization of
leases.
Answer D is correct because AU 508 states that such an opinion is expressed when the financial b) Information comes to the auditor’s attention that raises substantial doubt about the entity’s
statements taken as a whole are not presented fairly in conformity with generally accepted accounting ability to continue as a going concern.
principles. c) The chief executive officer refuses the auditor access t minutes of board of director’s
meetings.
d) Tests of controls show that the entity’s internal control structure is so poor that it cannot be
96. An auditor should disclose the substantive reasons for expressing an adverse opinion in an relied upon.
explanatory paragraph
a) Preceding the scope paragraph. Answer A is correct. Financial statements departing from GAAP, in this case a FASB Statement on the
b) Preceding the opinion paragraph. capitalization of leases, result in either a qualified "except for" opinion or an adverse opinion.
c) Following the opinion paragraph.
d) Within the notes to the financial statements. Answer B is incorrect. Substantial doubt about an entity's ability to remain a going concern may lead to
an explanatory paragraph being added to an unqualified opinion or a disclaimer of an opinion.
Answer A is incorrect. Placing an explanatory paragraph before the scope paragraph in order to disclose
the substantive reasons for expressing an adverse opinion is improper placement of such a paragraph. Answer C is incorrect. Scope limitations, in this case the restriction of access to board of directors'
minutes, result in either a qualified "except for" opinion or a disclaimer of opinion.
Answer B is correct. AU 508 requires that explanatory paragraphs disclosing the substantive reasons for
expressing an adverse opinion precede the opinion paragraph of the auditor's report. Answer D is incorrect. Internal control weaknesses will cause the auditor to perform additional
substantive procedures to obtain the required evidential matter. If the substantive procedures cannot
Answer C is incorrect. Placing an explanatory paragraph after the opinion paragraph in order to disclose provide the required evidential matter, a disclaimer of opinion is required.
the substantive reasons for expressing an adverse opinion is improper placement of such a paragraph.

Answer D is incorrect. Placing an explanatory paragraph within the notes to the financial statements in 99. A CPA who is associated with the financial statements of a public entity, but has not audited or
order to disclose the substantive reasons for expressing an adverse opinion is improper placement of reviewed such statements, should
such a paragraph. a) Insist that they be audited or reviewed before publication.
b) Read them to determine whether there are obvious material misstatements.
c) State these facts in the accompanying notes to the financial statements.
97. An auditor’s report includes the following statement: “The financial statements do not present d) Issue a compilation report.
fairly the financial position, results of operations, or cash flows in conformity with generally
accepted accounting principles.” This auditor’s report was most likely issued in connection with Answer A is incorrect because it is not the CPA's role to insist that financial statements be audited or
financial statements that are reviewed.
a) Inconsistent.
20
Audit Report
Answer B is correct because AU 504 states that when a CPA is associated with financial statements of a
public entity but has not audited or reviewed such statements, the CPA has no responsibility to apply any
procedures beyond reading the statements for obvious material misstatements. 102. Accepting an engagement to compile a financial projection for a publicly held company most
likely would be inappropriate if the projection were to be distributed to
Answer C is incorrect because while the financial statements should be conspicuously marked as a) A bank with which the entity is negotiating for a loan.
unaudited, no more detail is required. b) A labor union with which the entity is negotiating a contract.
c) The principal stockholder, to the exclusion of the other stockholders.
Answer D is incorrect because a compilation need not be performed. Also, compilations are primarily for d) All stockholders of records as of the report date.
nonpublic firms.
Answer A is incorrect. The bank is a party with which the company is negotiating directly, and therefore
distribution is appropriate.
100. The annual report of a publicly held company presents the prior year’s financial statements
which are clearly marked “unaudited” in comparative form with current year audited financial Answer B is incorrect. The labor union is a party with which the company is negotiating directly, and
statements. The auditor’s report should therefore distribution is appropriate.
a) Expresses an opinion on the audited financial statements and contain a separate
paragraph describing the responsibility assumed for the financial statements of the prior Answer C is incorrect. The principal stockholder is a party with which the company may be negotiating
period. directly, and therefore distribution may be appropriate.
b) Disclaim an opinion on the unaudited financial statements, add an explanatory paragraph
with respect to consistency, and express an opinion on the current year’s financial Answer D is correct because projections are limited use, not general use statements, and accordingly
statements. should only be distributed to the company and third parties with whom the company is negotiating
c) State that the unaudited financial statements are presented solely for comparative directly.
purposes and express an opinion only on the current year’s financial statements.
d) Express an opinion on the audited financial statements and state whether the audited
financial statements were complied or reviewed. 103. When an accountant is engaged to report on a nonpublic entity’s compiled financial statements
that omit substantially all disclosures required by generally accepted accounting principles, the
Answer A is correct because AU 504 states that when unaudited financial statements are presented in accountant should indicate in the compilation report that the financial statements are
comparative form with audited statements, the report on the current period should include as a separate a) Restricted for internal use only by the entity’s management.
paragraph an appropriate description of the responsibility assumed for the financial statements of the b) Not to be given to financial institution for the purpose of obtaining credit.
prior period. c) Complied in conformity with a comprehensive basis of accounting other than accepted
accounting principles.
Answer B is incorrect because there is no indication that the accounting principles have not been d) Not designed for those who are uninformed about the omitted disclosures.
consistently applied.
Answer A is incorrect because an internal use only restriction is not required.
Answer C is incorrect because the auditor may not restrict the use of the financial statements to
comparative analysis. Answer B is incorrect. This restriction is not required.

Answer D is incorrect because the statements were not compiled or reviewed; these forms of auditor Answer C is incorrect because the lack of such disclosures is not considered a comprehensive basis of
association are for nonpublic firms. accounting other than GAAP. See the outline of AU 623 for information on comprehensive basis financial
statements.

101. An accountant’s compilation report should be dated as of the date of Answer D is correct because AR 100 requires that the report indicate that the financial statements are not
a) Completion of fieldwork. designed for those who are uninformed about the omitted disclosures.
b) Completion of the compilation.
c) Transmittal of the compilation report.
d) The latest subsequent event referred to in the notes to the financial statements. 104. When an accountant is not independent of a client and is requested to perform a compilation of
its financial statements, the accountant
Answer A is incorrect. Refer to the correct answer explanation. a) Is precluded from accepting the engagement.
b) May accept the engagement and need not disclose the lack of independence.
Answer B is correct because AR 100 requires that the date of completion of the compilation be used as c) May accept the engagement and should disclose the lack of independence, but not the
the appropriate date for an auditor's compilation report. reason for the lack of independence.
d) May accept the engagement and should disclose both the lack of independence and the
Answer C is incorrect. Refer to the correct answer explanation. reason for the lack of independence.

Answer D is incorrect. Refer to the correct answer explanation. Answer A is incorrect because an accountant may accept the engagement.
21
Audit Report
107. In performing a compilation of financial statements of a nonpublic entity, the accountant decides
Answer B is incorrect because the lack of independence must be disclosed. that modification of the standard report is not adequate to indicate deficiencies in the financial
statements taken as a whole, and the client is not willing to correct the deficiencies. The
Answer C is correct because AR 100 states that an accountant may accept such an engagement and that accountant should therefore
the compilation report should disclose the lack of independence, but not the reason for the lack of a) Perform a review of the financial statements.
independence. b) Issue a special report.
c) Withdraw from the engagement.
Answer D is incorrect because the reason for the lack of independence is not to be disclosed. d) Express an adverse audit opinion.

Answer A is incorrect because a review is a form of association in which the client's refusal to correct the
105. How does an accountant make the following representations when issuing the standard report deficiencies will cause the same problems as their refusal in a compilation.
for the compilation of a nonpublic entity’s financial statements?
Answer B is incorrect because this is not a circumstance in which a special report is to be issued.
The financial statements have The accountant has compiled
not been audited the financial statements Answer C is correct because when an auditor performing a compilation believes that modification of the
A Implicitly Implicitly standard compilation report is not adequate to indicate the financial statements' deficiencies, the auditor
B Explicitly Explicitly should withdraw from the compilation and provide no further services with respect to the financial
C Implicitly Explicitly statements.
D Explicitly Implicitly
Answer D is incorrect because an audit has not been performed, and therefore, the auditor may not
Answer A is incorrect. Refer to the correct answer explanation. express an adverse opinion.

Answer B is correct. The report explicitly states that the financial statements have not been audited and
that the accountant has compiled them. 108. If an accountant submits compiled financial statements to a client that are reasonably expected
to be used by a third party, which of the following is correct?
Answer C is incorrect. Refer to the correct answer explanation. a) A compilation report is required.
b) No compilation report is required, but an engagement letter must indicate that the financial
Answer D is incorrect. Refer to the correct answer explanation. statements are to be used by a third party.
c) Notes describing the compilation must be included with the financial statements.
d) Compilations are not allowable under such a circumstance.
106. When reporting on compiled financial statements presented in conformity with the cash receipts
and disbursements basis of accounting that do not disclose the basis of accounting used, the Answer A is correct because when third party reliance on the compiled financial statements is expected, a
accountant should compilation report must be included.
a) Disclose the basis in the notes to the financial statements.
b) Clearly label each page “unaudited” Answer B is incorrect because a compilation report is required, and an engagement letter, while desirable
c) Disclose the basis of accounting in the accountant’s report. is not required.
d) Recompile the financial statements using generally accepted accounting principles.
Answer C is incorrect because no such notes describing a compilation are ever included in the financial
Answer A is incorrect because it is the responsibility of management, and not that of the accountant, to statements. Also, notes may be relating to the financial statements may be omitted if their omission is so
disclose the basis in the notes. indicated in the compilation report.

Answer B is incorrect because each page should be marked with a reference such as "See Accountant's Answer D is incorrect because compilations are allowable.
Compilation Report" also, as indicated, the basis should be included in the accountant's report.

Answer C is correct because AR 100 requires that when financial statements compiled in conformity with 109. Which of the following is correct relating to an accountant’s responsibility when submitting
a comprehensive basis of accounting other than generally accepted accounting principles do not include compiled financial statements to the management of a client that has informed the auditor that
disclosure of that basis, it should be disclosed in the accountant's report. the financial statements are for management use only?
a) The accountant must perform substantive tests of control to obtain assurance that use of
Answer D is incorrect because there is no requirement to recompile the financial statements using the financial statements will be so restricted.
generally accepted accounting principles. b) A restricted use compilation report must be included with the financial statements.
c) The accountant must document this restriction in an engagement letter.
d) Each page of the financial statements should have a restriction such as “Restricted for
Management’s Use Only”.

22
Audit Report
Answer A is incorrect because an accountant may rely on management's representation without further Answer D is incorrect. This statement would be included in a compilation report.
inquiry, unless information contradicting management's representation comes to the accountant's
attention.
112. Clark, CPA, compiled and properly reported on the financial statements of Green Co., a
Answer B is incorrect because no such restricted use compilation reports must be included with the nonpublic entity, for the year ended March 31, 2000. These financial statements omitted
financial statements. substantially all disclosures required by generally accepted accounting principles (GAAP).
Green asked Clark to compile the statements for the year ended March 31, 2001, and to include
Answer C is incorrect because the restriction may either be documented in an engagement letter or all GAAP disclosures for the 2001 statements only, but otherwise present both years’ financial
through issuance of a compilation report. statements in comparative form. What is Clark’s responsibility concerning the proposed
engagement?
Answer D is correct because the restriction must be indicated on each page of the financial statements. a) Clark may not report on the comparative financial statements because the 2000
statements are not comparable to the 2001 statements.
b) Clark may report on the comparative financial statements provided the 2000 statements
110. A CPA has audited financial statements and issued an unqualified opinion on them. do not contain any obvious material misstatements.
Subsequently the CPA was requested to compile financial statements for the same period that c) Clark may report on the comparative financial statements provided an explanatory
omit substantially all disclosures and are to be used for comparative purposes. In these paragraph is added.
circumstances the CPA may report on comparative compiled financial statements that omit such d) Clark may report on the comparative financial statements provided Clark updates the
disclosures provided the report on the 2000 statements.
a) Missing disclosures are immaterial in amount.
b) Financial statements and notes appended thereto are not misleading. Answer A is correct. The CPA may not report on the comparative financial statements because of a lack
c) Accountant’s report indicates the previous audit and the date of the previous report. of comparability.
d) Previous auditor’s report accompanies the comparative financial statements.
Answer B is incorrect. Refer to the correct answer explanation.
Answer A is incorrect because there is no requirement that allows missing disclosures if they are
immaterial. Answer C is incorrect. Refer to the correct answer explanation.

Answer B is incorrect. In circumstances where the auditor has been requested to compile financial Answer D is incorrect. Refer to the correct answer explanation.
statements which omit substantially all disclosures and are not to be used for comparative purposes, the
only requirement is that the financial statements and notes are not to be misleading.
113. When compiled financial statements are accompanied by a report, that report should state all of
Answer C is correct because a CPA may report on comparative compiled statements which omit the following except
substantially all disclosures to financial statements which were previously audited if an additional a) The accountant does not express an opinion or any other form of assurance on them.
paragraph indicates the previous audit and the date of the previous report. b) A compilation has been performed.
c) A compilation is limited to presenting in the form of financial statements information that is
Answer D is incorrect because there is no requirement that the previous auditor's report accompany the the representation of management.
comparative statements. d) A compilation consists principally of inquiries of company personnel and analytical
procedures applied to financial data.

111. Which of the following statements should not be included in an accountant’s standard report Answer A is incorrect because AR 100 requires that a compilation report express no opinion or any other
based on the compilation of an entity’s financial statements? form of assurance.
a) A statement that the compilation was performed in accordance with standards established
by the PICPA. Answer B is incorrect because AR 100 requires that a compilation report disclose that a compilation has
b) A statement that the accountant has not audited or reviewed the financial statements. been performed.
c) A statement that the accountant does not express an opinion but expresses only limited
assurance on the financial statements. Answer C is incorrect because AR 100 requires that a compilation report disclose that a compilation is
d) A statement that a compilation is limited to presenting, in the form of financial statements, limited to presenting in the form of financial statements information that is the representation of
information that is the representation of management. management.

Answer A is incorrect. This statement would be included in a compilation report. Answer D is correct because a review report, not a compilation report, states that a review consists
primarily of inquiries and analytical procedures.
Answer B is incorrect. This statement would be included in a compilation report.

Answer C is correct. A compilation report provides no assurance on the financial statements. 114. When a compilation report is being issued, each page of the financial statements should include
a reference such as
23
Audit Report
a) See accompanying accountant’s footnotes. b) Perform a compilation.
b) Unaudited, see accountant’s disclaimer. c) Perform an examination.
c) See accountant’s compilation report. d) Apply agreed-upon procedures.
d) Subject to compilation restrictions.
Answer A is correct. The professional standards which presents the standards with respect to the
Answer A is incorrect. Refer to the correct answer explanation. accountant's obligations for prospective financial statements (forecasts and projections), does not allow
for the review form of association under any circumstance.
Answer B is incorrect. Refer to the correct answer explanation.
Answer B is incorrect. The professional standards allows the results of prospective financial statement
Answer C is correct. Each page of compiled financial statements should include the reference "see compilation engagements to be provided to third parties.
accountant's compilation report."
Answer C is incorrect. The professional standards allows the results of prospective financial statement
Answer D is incorrect. Refer to the correct answer explanation. examinations to be provided to third parties.

Answer D is incorrect. An accountant may accept an engagement to apply agreed-upon procedures


115. A CPA who is not independent may issue a when third-party use of prospective financial statements is restricted to specified third-party users who
a) Compilation report. have participated in establishing the nature and scope of the engagement and who take responsibility for
b) Review report. the adequacy of the procedures.
c) Comfort letter.
d) Qualified opinion.
118. An auditor who was engaged to perform an audit of the financial statements of a nonpublic entity
Answer A is correct since AR 100 allows a compilation to be performed by a CPA who is not independent. has been asked by the client to refrain from performing various audit procedures and change the
nature of the engagement to a review of the financial statements in accordance with standards
Answer B is incorrect because an accountant is precluded from issuing a review report on the financial established by the AICPA. The client’s request was made because the cost to complete the
statements of an entity with respect to which the CPA is not independent. audit was significant. Under the circumstances the auditor would most likely
a) Qualify the auditor’s report and refer to the scope limitation.
Answer C is incorrect because independence is necessary for an auditor to issue letters to underwriters, b) View the request as an indication of a possible irregularity.
or "comfort letters." c) Complete the examination which was in progress.
d) Honor the client’s request.
Answer D is incorrect because a qualified opinion presupposes the performance of an audit; audits must
be performed by independent auditors. Answer A is incorrect. Refer to the correct answer explanation.

Answer B is incorrect. Refer to the correct answer explanation.


116. Which of the following should be included in an accountant’s standard report based upon the
review of a nonpublic entity’s financial statements? Answer C is incorrect. Refer to the correct answer explanation.
a) A statement that the review was performed in accordance with generally accepted review
standards. Answer D is correct because the auditor is to consider the reasonableness of the request and, when
b) A statement that a review consists principally of inquiries and analytical procedures. reasonable justification exists (e.g., significant cost savings), the client's request may be honored.
c) A statement that the accountant is independent with respect to the entity.
d) A statement that a review is substantially greater in scope that a compilation.
119. During a review of the financial statements of a nonpublic entity, the CPA finds that the financial
Answer A is incorrect because the term "generally accepted review standards" is not used in the report. statements contain a material departure from generally accepted accounting principles. If
management refuses to correct the financial statement presentations, the CPA should
Answer B is correct because a statement is included that a review consists principally of inquiries and a) Disclose the departure in a separate paragraph of the report.
analytical procedures. b) Issue an adverse opinion.
c) Attach a footnote explaining the effects of the departure.
Answer C is incorrect because the accountant does not state that s/he is independent. d) Issue a compilation report.

Answer D is incorrect because no comparison to a compilation is made in the report. Answer A is correct because the departure should be disclosed in a separate paragraph of the report if
the accountant concludes that modification is appropriate.

117. When third-party use of prospective financial statements is expected, an accountant may not Answer B is incorrect because adverse opinions are only appropriate for audits, not reviews.
accept an engagement to
a) Perform a review. Answer C is incorrect because it is not the role of a CPA to attach footnotes to a client's reports.
24
Audit Report
a) An uncertainty.
Answer D is incorrect because a review has been performed, not a compilation. b) Lack of consistency.
c) Reference to another accountant.
d) Inadequate disclosure.
120. Which of the following would not be included in a CPA’s report based upon a review of the
financial statements of a nonpublic entity? Answer A is incorrect because an uncertainty affecting interim financial information would not cause the
a) A statement that the review was in accordance with generally accepted auditing accountant to modify the review report.
standards.
b) Statements that all information included in the financial statements are the representations Answer B is incorrect because AU 722 states that a lack of consistency in the application of accounting
of management. principles affecting interim financial information will not cause the accountant to modify the review report.
c) A statement describing the principal procedures performed.
d) A statement describing the auditor’s conclusion based upon the results of the review. Answer C is incorrect because reference to another accountant is not considered a modification of the
report.
Answer A is correct because reviews, in contrast to audits of financial statements, are not made in
accordance with GAAS. Reviews are made in accordance with standards established by the AICPA Answer D is correct because departures from generally accepted accounting principles, which include
which are Statements on Standards for Accounting and Review Services. AR 100, Compilation and adequate disclosure, require modification of the accountant's report.
Review of Financial Statements, provides standards for performing reviews and compilations and issuing
reports thereon.
123. The objective of a review of interim financial information is to provide the CPA with a basis for
Answer B is incorrect because reports on reviews do state that the financial statements are a) Expressing a limited opinion that the financial information is presented in conformity with
representations of management. generally accepted accounting principles.
b) Expressing a compilation opinion on the financial information.
Answer C is incorrect because a report on a review does indicate that the review consists principally of c) Reporting whether material modification should be made to such information to make it
inquiries of company personnel and analytical procedures applied to the financial data. conform with generally accepted accounting principles.
d) Reporting limited assurance to the board of directors.
Answer D is incorrect because a report on a review does describe the accountant's conclusions based
upon the results of the review. Answer A is incorrect because while "limited assurance" is provided in a review, a review report is not
referred to as a "limited opinion."

121. An accountant who had begun an audit of the financial statements of a nonpublic entity was Answer B is incorrect because a compilation is an alternate form of accounting service in which the
asked to change the engagement to a review because of a restriction on the scope of the audit. accountant provides no express assurance on the statements.
If there is reasonable justification for the change, the accountant’s review report should include
reference to the Answer C is correct because AU 722 states that the objective is to provide the accountant with a basis for
reporting whether material modification should be made for such information to conform with generally
Original engagement that Scope limitation that caused to accepted accounting principles.
was agreed to changed engagement
A Yes Yes Answer D is incorrect because a review report need not be restricted to use only by the board of
B Yes No directors.
C No Yes
D No No
124. Each page of a nonpublic entity’s financial statements reviewed by an accountant should include
Answer A is incorrect. Refer to the correct answer explanation. the following reference:
a) See Accountant’s Review Report.
Answer B is incorrect. Refer to the correct answer explanation. b) Reviewed, No Accountant’s Assurance Expressed.
c) See Accompanying Accountant’s Footnotes.
Answer C is incorrect. Refer to the correct answer explanation. d) Reviewed, No Material Modification Required.

Answer D is correct. In such circumstances the CPA should neither include reference to the original Answer A is correct because AR 100 indicates that each page should include a reference such as, "See
engagement or to the scope limitation. Accountant's Review Report."

Answer B is incorrect because it suggests that no accountant's assurance is expressed, when review
122. A modification of the CPA’s report on a review of the interim financial statements of a publicly reports do provide limited assurance.
held company would be necessitated by which of the following?
Answer C is incorrect because the footnotes are those of management, not the accountant.
25
Audit Report
c) Price-level basis financial statements.
Answer D is incorrect because no indication as to the need for material modifications is to accompany d) Compliance with a contractual agreement not related to the financial statements.
each page of the financial statements.
Answer A is incorrect because AU 623 specifically excludes reports issued in connection with feasibility
studies from the definition of special reports.
125. The objective of a review of interim financial information of a public entity is to provide an
accountant with a basis for reporting whether Answer B is incorrect because the report on a review of interim financial information is specifically
a) A reasonable basis exists for expressing an updated opinion regarding the financial excluded from qualifying as a special report.
statements that were previously audited.
b) Material modification should be made to conform with generally accepted accounting Answer C is correct because a report issued in connection with price-level basis financial statements (a
principles. comprehensive basis of accounting other than GAAP) is considered a special report.
c) The financial statements are presented fairly in accordance with standards of interim
reporting. Answer D is incorrect because the report issued in connection with compliance with a contractual
d) The financial statements are presented fairly in accordance with generally accepted agreement unrelated to the financial statements is specifically excluded from qualifying as a special
accounting principles. report.

Answer A is incorrect because no updated opinion is being issued.


128. An auditor’s report would be designated as a special report when it is issued in connection with
Answer B is correct because AU 722 states that the objective of a review of interim financial information is which of the following?
to provide a basis for reporting on whether material modification should be made for such information to a) Financial statements for an interim period which are subjected to a review.
conform with generally accepted accounting principles. b) Financial statements which are prepared in accordance with a comprehensive basis of
accounting other than generally accepted accounting principles.
Answer C is incorrect because the CPA does not report on whether the statements are presented fairly in c) Financial statements which purport to be in accordance with generally accepted
accordance with standards for interim reporting. accounting principles but do not include a presentation of the statements of cash flows.
d) Financial statements which are unaudited and are prepared from a client’s accounting
Answer D is incorrect because the report issued does not indicate whether the financial statements are records.
presented fairly in accordance with generally accepted accounting principles.
Answer A is incorrect because a report issued in connection with a review of interim financial statements
is not considered a special report.
126. An auditor may report on condensed financial statements that are derived from complete audited
financial statements if the Answer B is correct because a report on financial statements prepared in accordance with a
a) Auditor indicates whether the information in the condensed financial statements is fairly comprehensive basis of accounting other than GAAP is considered a special report.
stated in all material respects.
b) Condensed financial statements are presented in comparative form with the prior year’s Answer C is incorrect because financial statements which purport to be in accordance with GAAP but do
condensed financial statements. not include the statement of cash flows must normally be accompanied by a qualified opinion.
c) Auditor describes the additional review procedures performed on the condensed financial
statements. Answer D is incorrect because financial statements which are unaudited but have been compiled by the
d) Condensed financial statements are distributed only to management and the board of auditor from the client's accounting records should be accompanied by a compilation report. This report
directors. is not considered a special report per AU 623.

Answer A is correct. A report may be issued when the information in the condensed financial statements
is fairly stated in all material respects. 129. When there has been a change in accounting principles, but the effect of the change on the
comparability of the financial statements is not material, the auditor should
Answer B is incorrect. Prior year condensed financial information is not necessary. a) Refer to the change in an explanatory paragraph.
b) Explicitly concur that the change is preferred.
Answer C is incorrect. The report need not indicate the nature of any additional procedures. c) Not refer to consistency in the auditor’s report.
d) Refer to the change in the opinion paragraph.
Answer D is incorrect. Condensed financial statements may be distributed publicly.
Answer A is incorrect. Refer to the correct answer explanation.

127. One example of a “special report,” as defined by Statements on Auditing Standards, is a report Answer B is incorrect. Refer to the correct answer explanation.
issued in connection with
a) A feasibility study. Answer C is correct because AU 420 states that in such circumstances a standard unqualified audit
b) A review of interim financial information. report should be issued.
26
Audit Report
Answer D is incorrect. Refer to the correct answer explanation. Answer B is incorrect because the auditor has already determined the effects of the titles as being not
suitable.

130. An auditor is reporting on cash basis financial statements. These statements are best referred Answer C is incorrect because a disclaimer should only be issued when the auditor is not expressing an
to in his opinion by which of the following descriptions? opinion on the financial statements. Here, the titles were found to be not suitable, and thus the CPA must
a) Financial position and results of operations arising from cash transactions. express his reservations in a qualified or adverse opinion.
b) Assets and liabilities arising from cash transactions, and revenue collected and expensed
paid. Answer D is incorrect because the financial statements and related footnotes are the representations of
c) Balance sheet and income statement resulting from cash transactions. management rather than the auditor. Thus, the auditor's reservations belong in the audit report rather
d) Cash balance sheet and the source and application of funds. than in the statements.

Answer A is incorrect. Refer to the correct answer explanation.


133. An auditor’s report on financial statements that are prepared in accordance with a
Answer B is correct because the preferable titles when reporting on cash basis financial statements are comprehensive basis of accounting other than generally accepted accounting principles should
assets and liabilities arising from cash transactions, and revenue collected and expenses paid. preferably include all of the following, except
a) Disclosure of the fact that the financial statements are not intended to be presented in
Answer C is incorrect. Refer to the correct answer explanation. conformity with generally accepted accounting principles.
b) An opinion as to whether the use of the disclosure method is appropriate.
Answer D is incorrect. Refer to the correct answer explanation. c) An opinion as to whether the financial statements are presented fairly in conformity with
the basis of accounting described.
d) An opinion as to whether the disclosed basis of accounting has been applied in a manner
131. When an auditor conducts an examination in accordance with generally accepted auditing consistent with the preceding period.
standards and concludes that the financial statements are fairly presented in accordance with a
comprehensive basis of accounting other than generally accepted accounting principles such as Answer A is incorrect because AU 623 states that the disclosure of the fact that statements are not
the cash basis of accounting, the auditor should issue a intended to be presented in conformity with GAAP is required.
a) Disclaimer of opinion.
b) Review report. Answer B is correct because no comment need be made concerning the appropriateness of the disclosed
c) Qualified opinion. method.
d) Special report.
Answer C is incorrect because AU 623 states that the auditor's report should include an opinion as to
Answer A is incorrect. A disclaimer of opinion is not necessary. whether the statements are presented fairly in conformity with the basis of accounting described.

Answer B is incorrect because an audit, not a review, has been performed. Answer D is incorrect because AU 623 states that the auditor's report includes an opinion as to whether
the disclosed basis of accounting has been applied consistently.
Answer C is incorrect. A qualified opinion is not necessary.

Answer D is correct because such comprehensive basis reports are considered "special reports"; the 134. When asked to perform an examination in order to express an opinion on one or more specified
opinion would be unqualified. elements, accounts, or items of a financial statement, the auditor
a) May not described auditing procedures applied.
b) Should advise the client that the opinion will result in a piecemeal opinion.
132. If the auditor believes that financial statements which are prepared on a comprehensive basis of c) May assume that the first standard of reporting with respect to generally accepted
accounting other than generally accepted accounting principles are not suitably titled, the auditor accounting principles does not apply.
should d) Should comply with the request only if they constitute a major portion of the financial
a) Modify the auditor’s report to disclose any reservations. statements on which an auditor has disclaimed an opinion based on an audit.
b) Consider the effects of the titles on the financial statements taken as a whole.
c) Issue a disclaimer of opinion. Answer A is incorrect because the special report on specified elements may describe the results of
d) Add a footnote to the financial statements which explains alternative terminology. applying agreed-upon procedures to one or more elements.

Answer A is correct because if the auditor believes that financial statements prepared per a Answer B is incorrect because piecemeal opinions are inappropriate and should not be issued in any
comprehensive basis other than GAAP are not suitably titled, he should modify his report to disclose any situation. A piecemeal opinion is a positive expression of opinion on certain identified items in the
reservations. Titles such as balance sheet, income statement, etc., generally only apply to financial financial statements for a client for which the auditor either disclaimed an opinion or expressed an
statements per GAAP. Financial statements per the cash basis or income tax basis should be suitably adverse opinion on the financial statements taken as a whole.
described.
27
Audit Report
Answer C is correct because reports on specified elements or accounts of a financial statement are 137. Kramer, CPA is auditing the financial statements of Jeffersonville in accordance with
special reports. The standards for special reports are outlined in AU 623. The general standards, the Government Auditing Standards. Kramer’s report on compliance with laws and regulations
standards of fieldwork, and the third and fourth standards of reporting are applicable. Since specified should contain a statement of
elements are not statements, the first standard of reporting regarding GAAP is not applicable.
Positive assurance Negative assurance
Answer D is incorrect because reporting on an element which constitutes a major portion of the financial A No Yes
statements on which an auditor has disclaimed an opinion based on an audit is a piecemeal opinion B Yes Yes
which is prohibited. C No No
D Yes No
135. Which of the following statements with respect to an auditor’s report expressing an opinion on a Answer A is incorrect. Refer to the correct answer explanation.
specific item on a financial statement is correct?
a) Materiality must be related to the specified item rather than to the financial statements Answer B is correct. The auditor's report should contain a statement of positive assurance on those items
taken as a whole. which were tested for compliance and negative assurance on those items not tested.
b) Such a report can only be expressed if the auditor is also engaged to audit the entire set
of financial statements. Answer C is incorrect. Refer to the correct answer explanation.
c) The attention devoted to the specified item is usually less than it would be if the financial
statements taken as a whole were being audited. Answer D is incorrect. Refer to the correct answer explanation.
d) The auditor who has issued an adverse opinion on the financial statements taken as a
whole can never express an opinion on a specified item in these financial statements.
138. Whenever special reports, filed on a printed form designed by authorities, call upon the
Answer A is correct. When issuing a special report on a specific financial statement item, the independent auditor to make an assertion that the auditor believes is not justified, the auditor
measurement of materiality must be related to that item. should
a) Submit a short-form report with explanations.
Answer B is incorrect since an auditor may report on a single item without performing a complete financial b) Reword the form or attaché a separate report.
statement audit. c) Submit the form with questionable items clearly omitted.
d) Withdraw from the engagement.
Answer C is incorrect; the materiality threshold is lower for a single item than it would be for an overall
audit causing the examination for a single item to be more extensive. Answer A is incorrect because a short-form report is not required and may not be appropriate (e.g., a
report on a comprehensive basis of accounting other than GAAP).
Answer D is correct only in cases in which such reporting would be tantamount to expressing a piecemeal
opinion. Answer B is correct because when a printed report form calls upon an independent auditor to make an
assertion that s/he believes is not justified in making, the form should be reworded or attached to a
separate report.
136. Reports are considered special reports when issued in connection with
a) Compliance with aspects of regulatory requirements related to audited financial Answer C is incorrect because the form must conform to the applicable reporting standards. The
statements. omission of questionable items will not relieve the auditor of reporting responsibilities.
b) Pro forma financial presentations designed to demonstrate the effect of hypothetical
transactions. Answer D is incorrect because the auditor would only withdraw from the engagement if the rewording or
c) Feasibility studies presented to illustrate an entity’s results of operations. separate report is unacceptable to the client.
d) Interim financial information reviewed to determine whether material modifications should
be made to conform with generally accepted accounting principles.
139. An accountant has been asked to compile the financial statements of a nonpublic company on a
Answer A is correct because reports based on compliance with aspects of regulatory requirements prescribed form that omits substantially all the disclosures required by generally accepted
related to audited financial statements are considered to be special reports. accounting principles. If the prescribed form is a standard preprinted form adopted by the
company’s industry trade association, and is to be transmitted only to such association, the
Answer B is incorrect. It is an example of a report that is not classified as a special report. accountant
a) Need not advice the industry trade association of the omission of all disclosures.
Answer C is incorrect. It is an example of a report that is not classified as a special report. b) Should disclose the details of the omission in separate paragraphs of the compilation
report.
Answer D is incorrect. It is an example of a report that is not classified as a special report. c) Is precluded from issuing a compilation report when all disclosures are omitted.
d) Should express limited assurance that the financial statements are free of material
misstatements.

28
Audit Report
Answer D is correct because compiled statements, even those prepared in accordance with appraised
Answer A is correct because AR 300 states that there is a presumption that the information required by a liquidation values, are considered compilations.
prescribed form is sufficient to meet the needs of the body that designed or adopted it; accordingly, there
is no need for that body to be advised of departures from GAAP.
142. Given one or more hypothetical assumptions, a responsible party may prepare, to the best of its
Answer B is incorrect because details of the omissions need not be provided. knowledge and belief, an entity’s expected financial position, results of operations, and changes
in financial position. Such prospective financial statements are known as
Answer C is incorrect because a compilation report may be issued in such circumstances. a) Pro forma financial statements.
b) Financial projections.
Answer D is incorrect because compilation reports provide no assurance that the financial statements are c) Partial presentations.
free of material misstatement. d) Financial forecasts.

Answer A is incorrect. Pro forma financial presentations are designed to demonstrate the effect of a
140. The term “special reports” may include all of the following, except reports on financial statements future or hypothetical transaction by showing how it might have affected the historical financial statements
a) Of an organization that has limited the scope of the auditor’s examination. if it had been consummated during the period covered by those statements.
b) Prepared for limited purposes such as a report that relates to only certain aspects of
financial statements. Answer B is correct. Financial projections are prospective financial statements that include one or more
c) Of a not-for-profit organization which follows accounting by business enterprises hypothetical assumptions.
organized for profit.
d) Prepared in accordance with historical cost/constant dollar accounting. Answer C is incorrect. Partial presentations are presentations that do not meet the minimum presentation
guidelines.
Answer A is correct because a scope limitation does not require a special report. Special reports are
appropriate for financial information based on (1) a comprehensive basis of accounting other than GAAP, Answer D is incorrect. Financial forecasts are prospective financial statements based on the responsible
(2) specified elements, (3) compliance with various agreements, and (4) prescribed forms. party's assumptions reflecting conditions it expects to exist and the course of action it expects to take.

Answer B is incorrect because such limited purpose reports are included under the "specified elements"
category of special reports. 143. A CPA’s report on an examination of a forecast should include all of the following except
a) A description of what the forecast information is intended to represent.
Answer C is incorrect because not-for-profit statements are based on a comprehensive basis of b) A caveat as to the ultimate attainment of the forecasted results.
accounting other than GAAP, and are thus considered a special report. c) A statement that the CPA assumes no responsibility to update the report for events
occurring after the date of the report.
Answer D is incorrect because historical cost/constant dollar reports are based on a comprehensive basis d) An opinion as to whether the forecast is fairly presented.
of accounting other than GAAP, and are thus considered a special report.
Answer A is incorrect because a forecast review report must describe the forecast information.

141. Auditors’ reports issued in connection with which of the following are generally not considered to Answer B is incorrect because a forecast report does indicate that actual results achieved during the
be special reports or special purpose reports? forecast period may vary from the forecast and that such variations may be material.
a) Specified elements, accounts, or items of a financial statement.
b) Compliance with aspects of contractual agreements related to audited financial Answer C is incorrect because the forecast report explicitly states that the CPA bears no responsibility to
statements. update the report.
c) Financial statements prepared in conformity with the price-level basis of accounting.
d) Compiled financial statements prepared in accordance with appraised liquidation values. Answer D is correct because a forecast examination report presents an opinion concluding whether the
forecast meets AICPA guidelines, not on whether the forecast is fairly presented.
Answer A is incorrect because AU 623 states that auditors' reports issued in connection with specified
elements, accounts, or items of a financial statement qualify as special reports.
144. When a CPA is associated with the preparation of forecasts, all of the following should be
Answer B is incorrect because AU 623 states that auditors' reports issued in compliance with aspects of disclosed except the
contractual agreements related to audited financial statements qualify as special reports. a) Sources of information.
b) Character of the work performed by the CPA.
Answer C is incorrect because AU 623 states that reports on audited financial statements which have c) Major assumptions in the preparation of the forecasts.
been prepared following alternative bases of accounting such as price-level basis qualify as special d) Probability of achieving estimates.
reports.
Answer A is incorrect because Ethics Rule 201 requires the disclosure of sources of information included
in the forecasts.
29
Audit Report
147. Which of the following is correct concerning an engagement to apply agreed-upon procedures?
Answer B is incorrect because Ethics Rule 201 requires the disclosure of the character of work performed a) A clear understanding of the terms of the engagement must be established through use an
by the CPA. engagement letter.
b) Independence of the CPA is not required.
Answer C is incorrect because Ethics Rule 201 requires the disclosure of major assumptions used in c) The procedures maybe as limited or as extensive as the CPAs desire ranging from a mere
preparing the forecasts. reading of the information to performing search and verification procedures.
d) Use of the report is restricted to the specified users.
Answer D is correct because Ethics Rule 201 indicates that a CPA should not vouch for the achievability
of forecasts. Answer A is incorrect because the engagement letter is not required for agreed-upon procedures of
special elements, accounts, or items of financial statements.

145. The party responsible for assumptions identified in the preparation of prospective financial Answer B is incorrect because agreed-upon procedure engagements require that the CPA be
statements is usually independent.
a) A third-party lending institution.
b) A client’s management. Answer C is incorrect because the specified users, not the CPA, must take responsibility for the
c) The reporting accountant. sufficiency of the agreed-upon procedures for their purposes.
d) The client’s independent auditor.
Answer D is correct because use of the report is restricted to the specified users. A statement of
Answer A is incorrect. A third-party lending institution is not considered a party normally responsible for restriction on the use of the report should be provided if the report is intended to be used solely by the
the assumptions identified in prospective financial statements. specified users.

Answer B is correct. The professional standards state that the party normally responsible for
assumptions identified in prospective financial statements is management. However, in limited 148. Which of the following procedures is most likely to be an appropriate procedure when performed
circumstances, a responsible party can be persons outside of the entity, such as a party considering as an agreed-upon procedures engagement under the attestation standards?
acquiring the entity. a) Evaluation of the competence or objectivity of another party.
b) Interpreting documents outside the scope of the practitioner’s professional expertise.
Answer C is incorrect. Reporting accountants are not considered parties normally responsible for the c) Obtaining an understanding about a particular subject.
assumptions identified in prospective financial statements. d) Performance of mathematical computations.

Answer D is incorrect. The client's independent auditor is not considered a party normally responsible for Answer A is incorrect because an agreed-upon procedures engagement is to present specific findings to
the assumptions identified in prospective financial statements. assist users in evaluating management's assertions about an entity's compliance with specified
requirements, not to evaluate the competence or objectivity.

146. Green, CPA, is requested to render an opinion on the application of accounting principles by an Answer B is incorrect because a CPA should not interpret any documents, rather, only use specific
entity that is audited by another CPA. Green may findings.
a) Not accept such an engagement because to do so would be considered unethical.
b) Not accept such an engagement because Green would lack the necessary information on Answer C is incorrect because an accountant may (1) compare procedures to written requirements, (2)
which to base an opinion without conducting an audit. discuss procedures to be applied with specified users, and/or (3) review relevant contracts or
c) Accept the engagement but should form an independent opinion without consulting with communication from specified users to report on management's assertions. In reporting to users, only
the continuing CPA. specific information will be presented.
d) Accept the engagement but should consult with the continuing CPA to ascertain all the
available facts relevant to forming a professional judgment. Answer D is correct because an accountant will perform mathematical computation in an agreed-upon
procedures engagement.
Answer A is incorrect because the CPA can accept such an engagement.

Answer B is incorrect because Green will have access to necessary information. 149. An accountant may accept an engagement to apply agreed-upon procedures to prospective
financial statements provided that
Answer C is incorrect because the CPA should consult with the continuing CPA. a) Use of the report is to be limited to the specified users involved.
b) The prospective financial statements are also examined.
Answer D is correct because AU 625 allows the CPA to accept such an engagement, but states that the c) Responsibility for the adequacy of the procedures performed is taken by the accountant.
CPA should consult with the continuing auditor to make certain that he or she has all of the relevant d) Negative assurance is expressed on the prospective financial statements take as a whole.
available facts to form a professional judgment.
Answer A is correct because the application of agreed-upon procedures results in a limited use report.

30
Audit Report
Answer B is incorrect because the auditor is not required to examine the prospective financial statements. Answer A is incorrect because audits generally do not report on accuracy.

Answer C is incorrect because the specified users, not the accountant, must take responsibility for the Answer B is incorrect because it is impossible for the auditor to audit subjective evaluations.
adequacy of the procedures.
Answer C is correct because governmental audits often require a determination of compliance with
Answer D is incorrect because the accountant will not provide negative assurance on the prospective prescribed regulations.
financial statements taken as a whole.
Answer D is incorrect because internal control is encompassed by an audit leading to expression of an
opinion only on the fairness of the financial presentation.
150. Which of the following forms of auditor association are possible relating to management’s
discussion and analysis (MD&A)?
153. An auditor notes reportable conditions in a financial statement audit conducted in accordance
Review Examination with Government Auditing Standards. In reporting on internal control, the auditor should state
A Yes No that
B Yes Yes a) Expressing an opinion on the entity’s financial statements provides no assurance on
C No Yes internal control.
D No No b) The auditor obtained an understanding of the design of relevant controls and determined
whether they have been placed in operation.
Answer A is incorrect. Refer to the correct answer explanation. c) The specified government funding or legislative body is responsible for reviewing internal
control as a condition of continued funding.
Answer B is correct because SSAE 8 (AT 700) provides for both review and examinations of MD&A. d) The auditor has not determined whether any of the reportable conditions described in the
report are so severe as to be material weakness.
Answer C is incorrect. Refer to the correct answer explanation.
Answer A is incorrect because while the report states that the purpose of the audit was not to provide
Answer D is incorrect. Refer to the correct answer explanation. assurance on internal control, the report does not state that no assurance is provided.

Answer B is correct because AU 801 requires a description of the scope of the auditor's work, stating that
151. Which of the following is not an assertion embodied in management’s discussion and analysis the auditor obtained an understanding of the design of relevant controls, determined whether those
(MD&A)? controls have been placed in operation, and assessed control risk. AU 801 provides guidance on the
a) Completeness. required contents of reports on internal control.
b) Consistency.
c) Occurrence. Answer C is incorrect because the specified government funding or legislative body is not generally
d) Rights and obligations. expected to be responsible for reviewing internal control as a condition of continued funding.

Answer A is incorrect. Refer to the correct answer explanation. Answer D is incorrect because the report must include a statement about whether the auditor believes
any of the reportable conditions described in the report are material weaknesses, and if they are, it must
Answer B is incorrect. Refer to the correct answer explanation. identify the material weaknesses noted.

Answer C is incorrect. Refer to the correct answer explanation.


154. Which of the following professional services would be considered an attest engagement?
Answer D is correct because the attestation standards on MD&A do not include an assertion for rights a) A management consulting engagement to provide accounting information system advice
and obligations. Those standards indicate that the four standards are completeness, consistency with the to a client.
financial statements, occurrence, and presentation and disclosure. b) An engagement to report on compliance with statutory requirements.
c) An income tax engagement to prepare tax returns.
d) The compliance of financial statements from client’s accounting records.
152. Governmental auditing often extends beyond examinations leading to the expression of opinion
on the fairness of financial presentation and includes audits of efficiency, economy, Answer A is incorrect. Refer to the correct answer explanation.
effectiveness, and also
a) Accuracy. Answer B is correct because a report on compliance with statutory requirements might be structured as
b) Evaluation. an attest engagement in which the required "written assertion" relates to such compliance (AT 101).
c) Compliance.
d) Internal control. Answer C is incorrect. Refer to the correct answer explanation.

Answer D is incorrect. Refer to the correct answer explanation.


31
Audit Report
157. In an audit in accordance with Government Auditing Standards an auditor is required to report
on the auditor’s tests of the entity’s compliance with applicable laws and regulations. This
155. When engaged to audit a not-for-profit organization in accordance with Government Auditing requirement is satisfied by designing the audit to provide
Standards, an auditor is required to prepare a written report on compliance with laws and a) Positive assurance that internal control tested by the auditor is operating as prescribed.
regulations that includes b) Reasonable assurance of detecting misstatements that are material to the financial
a) All material and immaterial instances of noncompliance with laws and regulations. statements.
b) All instances or indications of illegal acts that could result in criminal prosecution. c) Negative assurance that reportable conditions communicated during the audit do not
c) A description of all material weakness noted during the engagement. prevent the auditor from expressing an opinion.
d) An explanation of the inherent limitations of internal control. d) Limited assurance that internal control designed by management will prevent or detect
errors, irregularities, and illegal acts.
Answer A is incorrect. Immaterial instances of noncriminal acts need not be included in a report on
compliance with laws and regulations. Answer A is incorrect. Assurance on internal control is provided in the internal control report, not the
compliance report.
Answer B is correct. AU 801 requires that the report on compliance with laws and regulations include all
material instances of noncompliance, and all instances or indications of illegal acts which could result in Answer B is correct. The audit must be designed to provide reasonable assurance that the financial
criminal prosecution. statements are free of material misstatements resulting from violations of laws and regulations that have
a direct and material effect on the determination of financial statement amounts.
Answer C is incorrect. Material weaknesses are included in the report on internal control, not in the
compliance report. Answer C is incorrect. Assurance on internal control is provided in the internal control report, not the
compliance report.
Answer D is incorrect. An explanation of inherent limitations is included in the report on internal control,
not in the compliance report. Answer D is incorrect. Assurance on internal control is provided in the internal control report, not the
compliance report.

156. An auditor notes reportable conditions in a financial statement audit conducted in accordance
with Government Auditing Standards. In reporting on the internal control structure, the auditor 158. Jones, CPA, is auditing an entity’s financial statements in accordance with Government Auditing
should state that Standards. Jones should prepare a written report which describes
a) Expressing an opinion on the entity’s financial statements provides no assurance on a) The specific plan for obtaining audit evidence from all the applicable governmental
internal control structure. agencies.
b) The auditor obtained an understanding of the design of relevant policies and procedures, b) The scope of the auditor’s testing of compliance with laws and regulations and of internal
and determined whether they have been placed in operation. controls.
c) The specified government funding or legislative body is responsible for reviewing internal c) The auditor’s independence and lack of any personal biases or external influences
control as a condition of continued funding. regarding the client.
d) The auditor has not determined whether any of the reportable conditions described in the d) The list of internal controls tested and the results of the testing.
report are so severe as to be material weaknesses.
Answer A is incorrect because the auditor is not required to report how the audit evidence is to be
Answer A is incorrect because while the report states that the purpose of the audit was not to provide obtained.
assurance on the internal control structure, the report does not state that no assurance is provided (AU
801). Answer B is correct because Government Auditing Standards requires that auditors report on the scope
of their testing of compliance with laws and regulations and of internal controls.
Answer B is correct because AU 801 requires a description of the scope of the auditor's work, stating that
the auditor obtained an understanding of the design of relevant policies and procedures, determined Answer C is incorrect because, although the auditor must maintain the appearance and the mental
whether those policies and procedures have been placed in operation, and assessed control risk. AU attitude of independence, there is no specific requirement to issue a separate report.
801 provides guidance on the required contents of reports on the internal control structure.
Answer D is incorrect because the auditor does not separately report the internal controls tested and the
Answer C is incorrect because the specified government funding or legislative body is not generally outcome of the tests.
expected to be responsible for reviewing the internal control structure as a condition of continued funding.

Answer D is incorrect because the report must include a statement about whether the auditor believes
any of the reportable conditions described in the report are material weaknesses, and if they are, it must
identify the material weaknesses noted (AU 801).

32

Potrebbero piacerti anche